Physics Problems

Descripción completa

Views 557 Downloads 42 File size 2MB

Report DMCA / Copyright

DOWNLOAD FILE

Recommend stories

Citation preview

PROBLEMAS SELECCIONADOS DE FISICA Para entrenamiento de olimpiadas en Escuela Media Superior y para profundización en el nivel básico de Ciencias e Ingenierías

Dr. Raúl Portuondo Duany UPRM 2010

1

A mi Profesor de Física, Medel Pérez Quintana, maestro, colega y amigo.

Problemas Seleccionados de Física Ed.D. Raúl Portuondo Duany UPRM, Mayagüez, 2010, ©

2

INDICE

Página

Prólogo

5

Primera parte: Física Fragmentada

9

1a-Medidas y estimaciones. Problemas.

11

1b-Medidas y estimaciones. Soluciones.

12

2a-Cinemática. Problemas.

14

2b-Cinemática. Soluciones.

16

3a-Fuerzas. Problemas.

22

3b-Fuerzas. Soluciones.

25

4a-Conservación y variación de cantidades mecánicas. Problemas.

30

4b-Conservación y variación de cantidades mecánicas. Soluciones.

33

5a-Sólido Rígido. Problemas.

44

5b-Sólido Rígido. Soluciones.

47

6a-Hidrostática. Problemas.

57

6b-Hidrostática. Soluciones.

58

7a-Termodinámica I. Problemas.

60

7b-Termodinámica I. Soluciones.

63

8a-Física Molecular. Problemas.

70

8b-Física Molecular. Soluciones.

72

9a-Termodinámica II. Problemas.

81

9b-Termodinámica II. Soluciones.

83

10a-Oscilaciones. Problemas.

88

10b-Oscilaciones. Soluciones.

89

11a-Campo eléctrico. Problemas.

95

11b-Campo eléctrico. Soluciones.

98 3

12a-Circuitos. Problemas.

100

12b-Circuitos. Soluciones.

103

13a-Magnetismo. Problemas.

114

13b-Magnetismo. Soluciones.

115

14a-Optica. Problemas.

121

14b-Optica. Soluciones.

125

15a-Física Moderna. Problemas.

137

15b-Física Moderna. Soluciones.

139

Segunda parte: Física Integrada

146

16a-Problemas de Resortes y Algo Más.

148

16b-Problemas de Resortes y Algo Más. Soluciones.

151

17a-Problemas con Análisis y Cálculos Gráficos.

159

17b-Problemas con Análisis y Cálculos Gráficos. Soluciones.

164

18a-Problemas con Análisis de Simetría.

178

18b-Problemas con Análisis de Simetría. Soluciones.

180

19a-Problemas con Cálculo Diferencial o Integral.

188

19b-Problemas con Cálculo Diferencial o Integral. Soluciones.

193

Apéndice: Cantidades físicas fundamentales.

217

I-Magnitudes físicas

217

II-Definiciones y leyes

225

Tablas de definiciones y leyes

226

4

PRÓLOGO

El presente libro es una colección de problemas resueltos de Física, dividido en dos partes: I-Problemas de diferentes secciones de la Física, como suele dividirse la Física en los cursos corrientes. Esta primera parte (Física Fragmentada) es de nivel algebraico y trigonométrico. II-Problemas que integran diferentes secciones del curso (Física Integrada), o emplean un instrumento específico en cada capítulo que se usa de manera similar en todas las secciones de la Física: análisis de simetría, análisis numérico de gráficos, análisis que requieren de derivadas e integrales y de sus propiedades. En cada capítulo de esta parte pueden aparecer problemas de mecánica, o de calor, o de electromagnetismo, o de óptica, o mezclas de estas secciones. Los problemas del libro son una selección de los muchos usados por el autor durante más de veinte años para entrenar alumnos de enseñanza media superior para Olimpiadas de Física Internacionales (IPhO) e Iberoamericanas (OIbF). Unos pocos responden a estándares de cursos clásicos de enseñanza media, y la inmensa mayoría requiere de un pensamiento más profundo que el estándar (y algunos requieren de Cálculo Diferencial e Integral, en la segunda parte). Este no es un libro para estudiar desde cero los conceptos y leyes de los cursos regulares de Física, sino para ofrecer una ejercitación en el proceso de solucionar problemas de alguna complejidad. El libro es más bien un complemento para reforzar el estudio de la Física que se ofrece en los cursos regulares. El libro está destinado a: a)Estudiantes que terminan el nivel medio superior y se preparan para entrar en carreras de ciencias e ingenierías de universidades de alto nivel. b)Estudiantes de enseñanza media superior que comienzan a prepararse para participar en Olimpiadas de Física. (Las Olimpiadas Internacionales de Física para el nivel de grado 12 requieren de una ejercitación aún un poco más profunda que la ofrecida aquí). c)Profesores de enseñanza media superior que desean enriquecer su arsenal de problemas resueltos y profundizar en los métodos de solución de problemas físicos. d)Para estudiantes de College (ciclo básico universitario) pueden ser útiles los problemas tanto de la parte I como de la II. El estudiante debe intentar resolver los problemas por sí mismo, pero si siente que no puede, o que le faltan elementos, podrá pasar a los problemas resueltos. No obstante, las habilidades de la operatoria de solución las adquirirá sólo (a) si realiza los pasos algebraicos que a veces en el libro solamente se indican, pero no se desarrollan paso a paso, y (b) si hace el trabajo de cálculo 5

numérico (a veces extenso) para comprobar que la respuesta es la que ofrece el texto y ver las aproximaciones y redondeos que se hicieron. Lamentablemente, el libro no incorpora ejercicios experimentales: el trabajo experimental solamente puede realizarse en el laboratorio con los instrumentos apropiados. Es la base del trabajo de la Física, pues una teoría por sí sola, sin el correspondiente aval de la experiencia, es material muerto y ocioso. El estudiante que aspire a las Olimpiadas de Física y al trabajo profesional con esta ciencia deberá procurarse el entrenamiento experimental necesario: los montajes y mediciones que deben realizarse para contestar problemas físicos son a veces verdaderos problemas de ingenio. En el libro, el estudiante podrá hacerse una vaga idea de este ingenio experimental en la parte II, en la sección de análisis numéricos con gráficos, donde la sola obtención del gráfico que se da como dato es toda una tarea experimental. Las fuentes de algunos de estos problemas están distribuidas en una veintena de libros y manuales empleados por el autor durante veinte años, más problemas que son del propio autor. Las discusiones desarrolladas son todas del autor, no meras traducciones. Las fuentes utilizadas son: En inglés: -International Physics Olympiads: Edited by Waldemar Gorzkowski; Polish Academy of Science, 1990. -IPhO; http://www.jyu.fi/ipho/ -Feynman Lectures on Physics. Exercises; Addison-Wesley, 1964 -Optics; B. Rossi; Addison Wesly, 1957 -Waves and Oscillations; F.S.Crawford; Mc Graw Hill, N.Y., 1966 -Fundamental Physics; J. Orear; J. Wiley, 1964. En ruso: -Problemas de las Olimpiadas de Moscú; A.I. Buzdin, V.A. Ilin; Redacc: S.S. Krotov; Moscú, Ed. Naúka 1988. -Revista Kvant, Moscú. -Olimpiadas de Física de la URRS; I.Sh. Slobodetski, V.A. Orlov; Moscú, Ed Prosvishenie, 1982 -Un problema, dos problemas, …; A.I. Busdin, A.R. Zilverman, S.S. Krotov; Moscú, Ed. Naúka, 1990.

6

En español: -Problemas seleccionados de la Física Elemental; B.B. Bujovtsev, V.D. Krichenkov; Ed. Mir, Moscú, 1979. -Problemas de Física General; L.E. Irodov; Ed. Mir, Moscú, 1985. -Problemas de Física; dirigido por S. Kosel; Ed. Mir. Moscú, 1986. -Preguntas y Problemas de Física; L. Tarásov y A. Tarasova; Ed. Mir, Moscú, 1988. -Problemas de Física; dirigido por O.Ya. Savchenko; Ed. Mir, Moscú, 1989. -Física en ejemplos y problemas; E. Butilov, A. Bicov, A. Kondratiev; Ed. Mir, Moscú, 1991. -OIbF; http://oc.uan.edu.co/oibf/oibf.htm -Física PSSC; Ed. Reverté; México, 1968. -Mecánica; R. Portuondo, M. Pérez; Ed. Pueblo y Educación, Cuba, 1983. -Leyes de Conservación (Curso Facultativo); R. Portuondo; Poligráfico “Haydé Santamaría”, Cuba, 1989. -Introducción a la Física General; R. Portuondo; ENPES; Univ. de La Habana, 1990. -Introducción a la Teoría de la Relatividad; R. Portuondo; RUM, PR, 2008.

7

8

PRIMERA PARTE: FISICA FRAGMENTADA

En esta parte, los problemas de Física están agrupados por capítulos según el tema físico con el que se relacionan:1-Medidas y estimaciones (5 problemas); 2-Cinemática (9); 3-Fuerzas (9); 4Conservación y variación de cantidades mecánicas (12); 5-Sólido Rígido (11); 6-Hidrostática (5); 7-Termodinámica I (11); 8-Física Molecular (13); 9-Termodinámica II (9); 10-Oscilaciones (6); 11-Campo eléctrico (6); 12-Circuitos (13); 13-Magnetismo (6); 14-Óptica (10); 15-Física Moderna (9). En el Apéndice al final del libro se presentan las definiciones matemáticas de las cantidades físicas y sus significados. El estudiante encontrará también las ecuaciones físicas necesarias para resolver los problemas, subdivididas en ecuaciones de definiciones y ecuaciones de leyes. Las primeras son aplicables siempre, dentro de su marco de definición clásico (a velocidades próximas a las de la luz, algunas definiciones se transforman), pero las leyes son aplicables solamente dentro de determinadas condiciones, aún dentro de la física clásica: el estudiante deberá conocer las condiciones de aplicación de cada ley para utilizarlas solamente en las situaciones admisibles. Para resolver los problemas, el estudiante necesitará además tener habilidades operatorias de Algebra (incluyendo ecuaciones cuadráticas y sistemas de ecuaciones), Geometría y Trigonometría. Y, en general, deberá razonar finamente con los elementos físicos del problema. Los problemas de esta parte son, en general, problemas avanzados para el nivel de los grados 11 y 12. Constituyen un entrenamiento inicial para estudiantes de ese nivel que aspiren a participar en competencias internacionales de Física. Son adecuados también para estudiantes del nivel de College en Ciencias e Ingenierías.

9

10

1A-Medidas y estimaciones. Problemas.

1.1-Cinco relojes son puestos a prueba en un laboratorio. Exactamente al mediodía, de acuerdo con la señal mundial estándar de tiempo WWV, se leen los relojes en días sucesivos de una semana, y se obtienen las lecturas mostradas en la tabla. Ordene los cinco relojes de acuerdo con el valor relativo de sus lecturas como buenos medidores del tiempo, de mejor a peor. Justifique su respuesta. Reloj

Domingo

Lunes

Martes

Miércoles

Jueves

Viernes

Sábado

A

12:03:59

12:02:49

12:01:54

12:01:52

12:01:32

12:01:22

12:01:12

B

11:59:59

12:00:02

11:59:57

12:00:07

12:00:02

11:59:56

12:00:03

C

12:26:40

12:26:56

12:27:12

12:27:27

12:27:44

12:27:59

12:28:14

D

12:50:45

12:51:43

12:52:41

12:53:39

12:54:37

12:55:35

12:56:33

E

12:03:59

12:02:52

12:01:45

12:00:38

11:59:31

11:58:24

11:57:17

1.2-Un litro de aceite es derramado sobre un lago tranquilo. Si la mancha de aceite se expande uniformemente hasta producir una capa con el espesor justamente de una molécula, con moléculas adyacentes en contacto, estime el diámetro de la mancha de aceite. Asuma que las moléculas de aceite tienen un diámetro de 2 ×10-10 m.

1.3-Usted está volando de USA a Europa, a 45o de latitud. ¿Qué distancia debe viajar directamente al Este para adelantar su reloj seis horas? (Radio de la Tierra: 6400 km).

1.4-Teniendo en cuenta los movimientos de traslación y rotación de la Tierra, calcule el tiempo que la Tierra demora en dar una vuelta completa sobre sí misma.

1.5-Una persona de 1.75 m de altura, parada a la orilla del mar, nota que una lancha que se aleja perpendicularmente a la costa a 20 km/h demora unos 15 minutos en perderse tras el horizonte. La misma persona, acostada a nivel del mar, ve ocultarse el Sol tras el horizonte, pero si se para vuelve a ver el borde del Sol durante 10 s más hasta que se oculta nuevalmente. La persona se encuentra en la zona ecuatorial. Estime con estos datos el radio de la Tierra. 11

1B-Medidas y estimaciones. Soluciones.

1.1-El mejor reloj será el que muestre más consistentemente la constancia de su periodicidad. Si cada día se atrasa (o adelanta) exactamente lo mismo que el día anterior, bastará hacer la corrección de esa cantidad constante cada día para tener la hora exacta con toda confianza. Los mejores son D y E: el D se adelanta siempre 58 s de un día al siguiente y el E se atrasa siempre 67 s. Luego el C, que tiende siempre a adelantarse cada día, entre 15 s y 17 s, lo que no es muy caótico. Luego el B, que se adelanta o atrasa cada día entre 3 s y 10 s, lo cual ya es impredecible si irá hacia atrás o hacia adelante, pero las desviaciones no son muy grandes. Finalmente el A sufre variaciones diarias que pueden oscilar desde 70 s hasta 10 s, lo cual es muy grande comparado con los restantes relojes.

1.2-El volumen del litro de aceite será igual al volumen del disco de aceite expandido sobre el agua del lago: V = πR2h. De aquí: R = Haciendo el cálculo:

, donde V = 1L = 1×10-3 m3 y h = 2×10-10 m.

R = 1.3×103 m = 1.3 km

1.3- Un paralelo de la Tierra barre 360o de Este a Oeste. Las 24 horas del día distribuidas en 360o significa que cada 15o (un “uso horario”) cambian una hora física los relojes sobre los distintos territorios (salvo ligeras modificaciones geopolíticas locales). A la latitud de 45o la circunferencia del paralelo terrestre tiene un radio r = RT cos45o donde RT es el radio terrestre. Entonces: r = 4525 km. Para adelantar el reloj 6 horas, habrá que barrer un ángulo θ de 6×15 = 90o = π/2 rad. Si se hace por el paralelo de 45o de latitud N, el arco recorrido tendrá una longitud: s = rθ = 4525×π/2 = 7109 km

1.4-Cuando decimos que el día demora 24 horas estamos indicando que cada 24 horas (como promedio anual) el Sol pasa por el cenit (el punto más alto de la “trayectoria” del Sol respecto a nosotros). Pero éste no es el tiempo de una vuelta de la Tierra sobre sí misma. La Fig. 1.4S muestra la Tierra rotando alrededor del Sol y alrededor de sí misma (no está escala). Si de A a B transcurre un día, puede notarse que el observador P que en A miraba al sol en su cenit ha rotado algo más de 360o (una vuelta) para ver en su cenit al Sol 24 horas más tarde. 12

Asumiendo que la trayectoria de la órbita de la Tierra alrededor del Sol es casi circular, el ángulo Δθ extra rotado cada día (muy exagerado en la figura), multiplicado por 365 y ¼ de días, valdrá 360o pues la Tierra y el observador P estarán en la posición inicial de su órbita anual. Entonces:

Sol

Δθ Δθ P P Tierra

365.25 Δθ = 360

A

Y de aquí:

Fig. 1.4S

B

Δθ =0.986o ≈ 1o Esto es, la Tierra rota casi 361o cada día. Y como en esos 361o invierte 24 h, en la vuelta completa de 360o invertirá solamente: Δt = 24×(360/361) = 23.9 h = 23 h y 54 min Éste es el verdadero período de rotación de la Tierra (que mide la duración del día respecto a las estrellas fijas, o día sideral), no el día solar de 24 horas. Podrá darse cuenta que en el año de 365 días, la Tierra da 366 vueltas sobre sí misma.

1.5-Si la lancha desaparece en unos 15 minutos (o ¼ hora), el hombre parado en la orilla del mar ve el horizonte a: s = vt = 5 km (distancia de A a B en la Fig. 1.5S). La visión del borde del Sol estando acostado se logra con una visual tangente a la esfera terrestre desde su punto de observación (A en la Fig. 1.5S), y la vision del borde del Sol ya parado se tapa en el punto B distante unos 5 km de A. Esta distancia es la rotada por la superficie terrestre en los 10 s que transcurrieron entre los ocultamiento del Sol estando acostado y parado. Teniendo en cuenta que la Tierra da una vuelta en casi 24 h, u 86 400 s, en 10 s la Tierra rotará: θ = (10/86 400)×2π = 7.2×10-4 rad.

Sol

s

A

B R

θ

Sol Tierra

ecuador

Fig. 1.5S

Pero s = θR, donde R es el radio terrestre, por lo que R = s/θ = 5 km / (7.2×10-4) = 6.9×103 km, que es próximo al valor aceptado de 6.4×103 km.

13

2A-Cinemática. Problemas.

2.1-Un helicóptero despega de una pista verticalmente con aceleración a =3.0 m/s2 y velocidad inicial vo= 0. Después de cierto tiempo t1 el piloto apaga el motor. El sonido del motor en el lugar del despegue deja de ser escuchado al cabo de un tiempo t2= 30 s después que despegó. Determine la velocidad del helicóptero en el momento de apagar el motor. Considere la velocidad del sonido u = 320 m/s.

2.2- Una carretera recta y una línea de ferrocarril son paralelas una a otra. Un carro de carreras está en reposo en un punto de la carretera y un tren de 100 m de largo está pasando junto a él a 81.0 km/h. Justo cuando pasa al lado del carro el último vagón, el carro arranca en la misma dirección del tren, con una aceleración constante de 4.0 m/s2. a)¿Al cabo de cuánto tiempo pasa el carro a la locomotora, asumiendo que todo el rato mantiene la misma aceleración? b)¿Qué distancia ha recorrido el carro en ese tiempo? c)¿Qué velocidad tiene el carro en el instante en que pasa a la locomotora? d)¿Cuál fue la mayor distancia que hubo entre el carro y el frente de la locomotora antes de que el carro pasara a la locomotora?

2.3-Tres partículas se encuentran en los vértices de un triángulo equilátero de lado l y se mueven con rapidez constante, v, de tal modo que siempre la primera se dirige en dirección a la segunda, la segunda hacia la tercera y la tercera hacia la primera. ¿Qué tiempo demoran en encontrarse en el centro del triángulo?

2.4-Dos proyectiles se lanzan simultáneamente desde el mismo punto, ambos en dirección horizontal, pero en sentidos opuestos, con rapideces iniciales v1 y v2. ¿Qué distancia los separa en el instante en que sus velocidades son mutuamente perpendiculares?

2.5-Cierta estructura articulada consta de tres rombos cuyos lados están en la relación 3:2:1 (Fig. 2.5: próxima página). La bisagra A3 se desplaza horizontalmente con velocidad vo. Determine las velocidades de las bisagras A1, A2 y B1 en el instante cuando todos los ángulos de la estructura son rectos. 14

A’

A B1 B2

O’

B3 A0

A1

A2

A3

vo

θ O

Fig. 2.5

B

Fig.2.6

B’

2.6-Dos pequeños anillos O y O’ se encuentran en varillas verticales fijas AB y A’B’. Un hilo inextensible amarrado al punto A’ y al anillo O pasa a través del anillo O’ (ver Fig. 2.6). Considerando que el anillo O’ se mueve hacia abajo a velocidad v1, determine la velocidad v2 del anillo O cuando μ2. El ángulo de inclinación del plano es tal que los cuerpos resbalan juntos por el plano hacia abajo.

m2 m1

θ

Fig. 3.1

a)Calcule la fuerza F con que el cuerpo 2 empuja al 1. b)Calcule bajo qué ángulo de inclinación θA del plano los cuerpos resbalan con rapidez constante.

3.2-En el sistema de la Fig. 3.2, la viga vertical A puede deslizar hacia abajo verticalmente sin fricción con la cuña B, ni con las paredes del orificio vertical por el que desciende. La cuña tiene un ángulo de inclinación θ. La relación de masas de la viga y la cuña es mA / mB = β. Calcule la aceleración vertical aA con que desciende la viga y la aceleración horizontal aB con que se mueve la cuña.

A

θ

B

Fig. 3.2 3.3-En el extremo de una tabla de longitud L y masa M se encuentra un pequeño bloque de masa m (Fig. 3.3, próxima página). La tabla puede deslizarse sin fricción sobre una superficie horizontal. El coeficiente de fricción entre el bloque y la tabla es μ. ¿Qué velocidad mínima vo hay que comunicar a la tabla con un empujón instantáneo para que ella se deslice bajo el bloque y logre quitárselo de arriba? 22

L

m

M

vo

Fig. 3.3 3.4-En el extremo de un resorte vertical de masa despreciable se cuelga una pesa de masa m. Seguidamente se cuelga otra pesa de igual masa en el medio del resorte ya estirado. Determinar la longitud del resorte estirado con las dos pesas. Considere conocidas la constante elástica del resorte, k, y su longitud natural, lo.

3.5-Un acróbata que se halla sobre la superficie lateral de un cilindro, mueve las piernas y desplaza al cilindro con velocidad constante (Fig. 3.5). El cilindro está sobre una superficie horizontal rugosa. Considerando que el coeficiente de fricción entre las zapatillas y el cilindro es μ, determine el ángulo límite θo que puede formar con la vertical el radio del cilindro que pasa por el punto en que se encuentra el acróbata. ¿Cuál será la fuerza de fricción entre el cilindro y el acróbata en este caso? La masa del acróbata es m.

corre hacia acá

θo

v constante rueda hacia acá

Fig. 3.5 3.6-Las masas de dos estrellas son m1 y m2 y la distancia ente ellas es l. Calcule el período de rotación de estas estrellas por órbitas circulares alrededor de su centro de masas.

3.7-Desde los polos Sur y Norte de la Tierra se lanzan simultáneamente dos cohetes con las mismas velocidades iniciales, horizontalmente dirigidas. Después de un tiempo τ = 3 h 20 min los cohetes están a la máxima separación uno del otro. Determine la distancia máxima d entre los 23

cohetes. La aceleración de caída libre sobre la superficie terrestre se asume conocida. El radio de la Tierra es Ro = 6 400 km.

3.8-Dos esferitas de masas m1 = 56 g y m2 = 28 g se cuelgan mediante dos hilos de longitudes l1 = 7 cm y l2 = 11 cm al extremo inferior de una barra que cuelga libremente (Fig. 3.8). Determine la velocidad angular ω con que debe rotar la barra alrededor de la vertical que pasa a lo largo de sí misma para no separarse de dicha posición vertical.

θ1 θ2

l1 m1

l2 m2

Fig. 3.8

eje

3.9-Un péndulo está formado por una varilla ligera con una esfera pequeña y muy densa en su extremo inferior. A su extremo superior se acopla un casquillo cilíndrico ligero de radio interior r que envuelve a un eje horizontal en rotación, accionado por un motor (Fig.3.9). La longitud desde el centro de la esferita hasta el centro del casquillo es l. El coeficiente de fricción cinética entre el eje y el casquillo es μ. Determine el ángulo de inclinación de la varilla respecto a la vertical cuando quede en equilibrio. (Considere toda la masa del sistema concentrada en el centro de la esferita).

A r

l

m

Fig. 3.9

24

3B-Fuerzas. Soluciones.

3.1-La Fig. 3.1S muestra las fuerzas sobre cada cuerpo y las direcciones tomadas como positivas. Planteando la segunda ley de Newton por componentes a cada cuerpo, y recordando que f =μN, tendremos:

N2 N1 F21

m1gsenθ + F21 – μ1N1 = m1a1

m2

f2

F12

m1

(+) f1 m2g

N1 – m1gcosθ = 0 θ

m2gsenθ – F12 – μ2N2 = m2a2

m1g (+)

Fig. 3.1S

N2 – m2gcosθ = 0

Ahora: F12 = F21 por acción y reacción, y les llamaremos simplemente F, que es el valor de la fuerza del bloque 2 sobre el 1. Como los cuerpos resbalan juntos: a1 = a2 ≡ a. Las ecuaciones pueden reducirse a: m1gsenθ + F – μ1m1gcosθ = m1a (1)

y m2gsenθ – F – μ2m2gcosθ = m2a

(2)

Sumando miembro a miembro (1) y (2), y despejando a: a = g[senθ (m1 + m2) – cosθ (μ1m1 + μ2m2)] / (m1 + m2)

(3)

Sustituyendo en (1) y despejando: F = m1 m2 g [(μ1 − μ2) / (m1 + m2)] cosθ b)Si la rapidez es constante, la aceleración en (3) la hacemos cero y obtenemos: tanθ = (μ1 m1 + μ2 m2) / (m1 + m2). De aquí: θ = tan-1[(μ1 m1 + μ2 m2) / (m1 + m2)]

3.2-En la Fig. 3.2S se muestra las fuerzas que influyen en los movimientos analizados, y se indica la relación geométrica de los desplazamientos en x y y de los cuerpos.

A F

Tomando positiva la dirección vertical hacia abajo: mAg – Fcosθ = mAaA

mAg

(1) θ

Tomando positivo hacia la derecha: Fsenθ = mBaB

θ Δy Δx

(2)

Fig. 3.2S 25

F

B

Además: Δy = Δx tanθ. Se desprende de aquí que: vA = vB tanθ Y sus variaciones en el tiempo darán las aceleraciones: aA = aB tanθ

(3)

Despejando aB de (1), (2) y 3), y arreglando, se obtiene: aB = g β /(cotθ + β tanθ) Y sustituyendo en (3): aA = = g β / (cot2θ + β)

3.3-La velocidad instantánea que se comunique a la tabla respecto a la superficie horizontal es la misma que adquiere el pequeño bloque respecto a la tabla. Lo que se pide es que esa velocidad vo alcance al bloquecito para recorrer la distancia L y quedar casi en reposo cuando esté a punto de salir de la tabla: 0 = vo2 – 2arel L

(1)

Aquí arel es la aceleración del bloquecito respecto a la tabla, que a su vez está acelerada respecto al piso. La fricción entre la tabla y el bloquecito es f = μmg, y produce en la tabla una aceleración aM de frenado hacia la izquierda respecto al piso (Fig. 3.3S), y una aceleración de impulsión hacia la derecha, am, sobre el bloquecito (ambas aceleraciones respecto al piso, que es el sistema inercial). Aplicando la segunda ley de Newton a cada cuerpo, en valores absolutos: m

M

f, am

v

f, aM

Fig. 3.3S MaM = μmg

L

mam = μmg

y

De estas ecuaciones: aM = μmg/M

y am = μg

(valores absolutos)

Asumiendo positiva la dirección hacia la derecha: aM = −μmg/M

y am = μg

La aceleración relativa del bloquecito respecto a la tabla es arel = am – aM = μg (1 + m/M) Sustituyendo en (1) y despejando: vo =

26

3.4-Con el peso mg colgado al extremo inferior del resorte, éste se estira mg/k, lo que significa que cada mitad se estira mg/2k. Si del extremo inferior no se cuelga más peso, la mitad inferior no se estirará más, pero si de la mitad superior se cuelga otra carga mg, esto equivaldrá a duplicar la tensión en la mitad superior del resorte, por lo que esa mitad se estirará el doble: 2(mg/2k) = mg/k. El alargamiento total será entonces el de la mitad superior más el de la mitad inferior: mg/k + mg/2k = (3/2) mg/k. La longitud total de resorte será entonces: l = lo + (3/2) mg/k.

3.5-Como el cilindro se mueve con velocidad constante es entonces un sistema inercial y podemos analizar el equilibrio del hombre respecto al cilindro con las leyes de Newton. Considerando al hombre como partícula, el diagrama de fuerzas sobre él se muestra en la Fig. 3.5Sa. Como el hombre se mantiene en esa posición respecto al cilindro, la suma de fuerzas sobre él es nula, estando sostenido por la fricción estática máxima, fs,m, si está ya en el ángulo para el cual el resbalón es inminente:

N

fs,m θo θo

fs,m – mg senθo = 0 y N – mg cosθo = 0.

mg

Además: fs,m = μN

Fig. 3.5Sa

De estas tres ecuaciones resulta: tanθo = μ, por lo que: θo = tan-1 μ.

μ

θo

Entonces: f = μN = μmg cosθo 1

El valor obtenido para la tangente permite plantear el triángulo auxiliar mostrado al lado, del cual se deduce el valor del coseno:

Fig. 3.5Sb

cosθo = 1 / Entonces: f = μmg/

3.6-Las estrellas se atraen gravitacionalmente con fuerzas iguales y opuestas. La fuerza de gravitación que experimenta cada estrella actúa como fuerza centrípeta sobre ella. Las estrellas deben describir circunferencias con centro común en el centro de masas del sistema: las estrellas estarán siempre en lados opuestos respecto al centro de masas, por lo que las frecuencias de rotación de ambas deben ser iguales. 27

Sea un sistema de referencia inercial con su origen en el centro de masas del sistema. Sean l1 y l2 las distancias desde cada estrella al origen del sistema, y l, la distancia entre las estrellas (se cumple: l = l1 + l2). Entonces: Gm1m2/ l2 = m1ω2l1

y

Gm1m2/ l2 = m2ω2l2

Simplificando términos: Gm2/ l2 = ω2l1

Gm1/ l2 = ω2l2

y

Sumando miembro a miembro: G(m1 + m2) / l2 = ω2( l1 + l2) = ω2l De aquí: ω2 = G (m1 + m2 ) / l3 . Pero ω = 2π/T. Entonces: T =

3.7-Sea un sistema de referencia con su origen en el centro de la Tierra y sus ejes orientados hacia las estrellas fijas (el sistema no rota con la Tierra). Los tipos de órbitas que siguen los cohetes lanzados desde los polos son elipses como se muestran en la Fig. 3.7S. Son elipses iguales cuyos ejes mayores valen L; la separación entre los vértices opuestos es: d = L + (L – 2Ro) = 2L – 2Ro

L

(1)

Tierra

y esta es la máxima separación posible entre los cohetes. L

Para determinarla podemos hacer uso de la tercera ley de Kepler si comparamos estas órbitas con otra conocida.

Fig. 3.7S

El período de estas órbitas es conocido, a saber, el doble del tiempo η en que recorrieron la mitad de esas órbitas. Según la tercera ley de Kepler hay una proporcionalidad entre el cuadrado del período y el cubo del semieje mayor, que para estos cohetes sería L/2. La órbita de parámetros conocidos a que podemos recurrir para completar la proporcionalidad es la de un satélite que da la vuelta a la Tierra pegado a su superficie, cuyo semieje mayor sería el propio radio terrestre, Ro, y su período es fácilmente calculable por la relación: g = ω2Ro = 4π2Ro/T2 , de donde T = )]2 = [(L/2) / Ro]3.

Entonces, por la tercera ley de Kepler: [2η / (

Despejando L y evaluando parámetros: L ≈ 5.6 Ro. Sustituyendo en (1): d = 9.2 Ro ≈ 5.9×104 km. 28

3.8-Sea un sistema de referencia fijo al techo del cual cuelga la barra. Apliquemos la segunda ley de Newton por componentes vertical y horizontal, con la condición de que la barra permanezca vertical. Esto implicará que las dos esferitas describan circunferencias con un centro común, en la vertical de la barra: la fuerza resultante sobre cada una de estas esferitas será una fuerza centrípeta. Si llamamos θ1 y θ2 al ángulo de cada hilo con la vertical, tendremos: Para la de masa m1:

T1 cosθ1 – m1g = 0

T1 senθ1 = m1ω2(l1 senθ1)

Para la de masa m2:

T2 cosθ2 – m2g = 0

T2 senθ2 = m2ω2(l2 senθ2)

Además, la resultante de fuerzas horizontales sobre la barra debe ser cero: T1 senθ1 = T2 senθ2 Se obtiene así un sistema de cinco ecuaciones con cinco incógnitas: T1, T2, θ1, θ2 y ω. ≈ 14 rad/s2

Despejando:

μN

mg

eje θ

θ A

r

N

3.9-La incógnita a determinar es el ángulo υ de la Fig. 3.9S. En el equilibrio, la línea de acción del peso debe pasar por el punto de suspensión del sistema, y la fuerza total hacia arriba debe compensar al peso del sistema. Entre la normal N y la vertical hay un ángulo θ en el punto de suspensión A en el equilibrio. En tal situación: mg cos θ = N

y

mg sen θ = μN

Dividiendo miembro a miembro la segunda ecuación por la primera: tan θ = μ (1) Además, de los triángulos de la figura: l sen υ = r sen θ

l θ

por lo que: sen θ = (l/r) sen υ

(2)

y: tan θ = sen θ / cos θ = sen θ / (1 – sen2 θ)1/2 O: tan θ = 1/(sen-2θ – 1)1/2

(3)

Sustituyendo (1) y (2) en (3): μ = 1 / [(l/r)-2 sen-2υ – 1]1/2 Fig. 3.9S

mg

Despejamos sen υ: sen υ = (r/l) μ/ (μ2 + 1)1/2 Y de aquí: υ = arc sen [(r/l) μ/ (μ2 + 1)1/2]

29

4A-Conservación y variación de cantidades mecánicas. Problemas.

4.1-Sobre una superficie horizontal lisa, junto a una pared, descansa un bloque simétrico de masa m1 con una oquedad de forma semiesférica, de radio r (Fig. 4.1). Desde la posición inicial indicada resbala un pequeño bloque de masa m2, sin fricción. Encuentre la velocidad máxima del bloque grande en su movimiento.

m2

r

m1

Fig. 4.1

4.2-Dos cuñas móviles idénticas de masa M poseen transiciones suaves con el plano horizontal (Fig. 4.2).

m

h

Desde la cuña izquierda resbala un cubito de masa m desde una altura h. ¿Qué máxima altura hmax se eleva el cubito sobre la cuña derecha? Desprecie la fricción.

M

M

Fig. 4.2

4.3-Sobre un grupo de tres cubos lisos idénticos que descansan sobre una superficie horizontal lisa (Fig. 4.3) incide un cilindro liso con velocidad v. La masa de cada cubo es igual a la masa del cilindro. El diámetro del cilindro y su altura son iguales al lado de cada cubo.

Vista superior

m m m

v

m

Fig. 4.3

Determine la velocidad de cada cuerpo después del choque.

4.4-Tres pequeños cuerpos cuyas masas se relacionan como 3:4:5 se mantienen en tres puntos diferentes de la superficie interior lisa de una taza semiesférica de radio r. El punto más bajo de la taza descansa en un plano horizontal. En cierto momento los cuerpos se sueltan y se aban30

donan a sí mismos. Sabiendo que la masa del más pequeño es m, calcule la máxima cantidad de calor Q que puede desprenderse de tal sistema. ¿Para qué posiciones iniciales de los cuerpos esto ocurre? Considere que todos los choques son absolutamente plásticos.

4.5-Sobre un apoyo de altura h = 5 m se halla una esfera de masa M = 200 g. Una bala de masa m = 10 g que avanza en dirección horizontal con velocidad v = 500 m/s perfora la esfera exactamente en la dirección de su diámetro horizontal. ¿A que distancia l cae al suelo la bala si la esfera cae a una distancia L = 20.0 m a partir de la base de la mesa? ¿Qué parte de la energía cinética de la bala se transforma en energía interna cuando la bala perfora a la esfera? Despreciar la resistencia del aire.

4.6-Una esfera rígida de masa 2m se dispara verticalmente hacia arriba con velocidad vo. A la esfera está ligado un hilo ligero de longitud l (l < vo2/ 2g) perfectamente inextensible. En el otro extremo del hilo hay ligada otra esfera rígida de masa m. El lanzamiento de la primera esfera ocurre prácticamente desde el mismo punto donde se encuentra la segunda. ¿Después de cuánto tiempo y a qué distancia del punto de lanzamiento chocan las bolas? Despreciar los diámetros de las esferas.

4.7-Un péndulo matemático se inclinó un ángulo de 90º respecto a la vertical y se soltó. En el momento en que el péndulo pasaba por su posición de equilibrio, su punto de suspensión comenzó a moverse hacia arriba con aceleración a. ¿Cuál será el máximo ángulo que se separará el péndulo de la vertical?

u

R

w v

Luna h

Fig. 4.8

4.8-Una nave de 1000 t (toneladas métricas) está en órbita circular alrededor de la Luna a 100 km sobre su superficie. Para acercarse casi rasante a la Luna, expulsa instantáneamente un chorro de gas hacia delante en su órbita, de modo que frena un poco y pasa a una nueva órbita que le permite pasar rasante a la Luna por un punto diametralmente opuesto al punto en que lanzó la masa de gas (Fig. 4.8).

El gas es expulsado con una velocidad de 10000 m/s respecto a la nave ¿Qué cantidad de gas expulsó la nave en la maniobra? (Radio de la Luna: 1700 km; aceleración de la gravedad en la superficie lunar: 1.7 m/s2).

31

4.9-Una partícula es impulsada horizontalmente con velocidad vo dentro de un casco semiesférico liso de radio R, en contacto con la superficie interior, desde un punto localizado en el borde del casco (Fig. 4.9). Calcule la altura h que descenderá la partícula antes de volver a subir por el casco.

R

vo

h

Fig. 4.9

4.10-Una cinta transportadora de longitud l se mueve con velocidad v respecto al suelo. Por su extremo derecho se introduce un bloque de masa m que se moverá en contra de la cinta (ver Fig. 4.10). Determine la velocidad vo (respecto al suelo) con la cual es necesario impulsar el bloque contra el movimiento de la cinta para que la cantidad de calor liberada por el frenado del bloque por la cinta sea la máxima posible. Considere conocido el coeficiente de fricción μ entre la cinta y el bloque. Determine la cantidad de calor liberada en el proceso de frenado sobre la cinta. (En la situación planteada se cumple que v < (2μlg)1/2).

l

vo

m μ

v

Fig. 4.10

4.11-Una varilla sin peso, de longitud l, con una bolita de masa m en su extremo superior, está articulada por el punto inferior A como indica la Fig.4.11, y se encuentra en posición vertical, rozando una caja de masa M sobre un piso sin fricción. Producto de un ligero golpecito el sistema se pone en movimiento. ¿Bajo qué relación de masas M/m la varilla formará un ángulo θo= π/6 con la horizontal en el momento en que se despegue de la caja? ¿Cuál es la velocidad de la caja en ese instante? Se desprecia cualquier fricción.

m M

l A

Fig. 4.11

4.12-Un cuerpo de masa m = 0.5 kg está unido con dos paredes verticales mediante dos resortes ligeros y realiza oscilaciones moviéndose rectilíneamente por una superficie horizontal (Fig. 4.12). Las amplitudes de dos elongaciones sucesivas respecto al punto de equilibrio a la derecha 32

y a la izquierda son A1= 10 cm y A2= 7 cm. Determine el coeficiente de fricción cinética μ entre el cuerpo y la superficie sabiendo que la constante elástica de ambos resortes es k = 15 N/m. k

k m

Fig. 4.12

4B-Conservación y variación de cantidades mecánicas. Soluciones.

4.1-Puesto que todas las superficies son lisas, no habrá pérdidas de energía mecánica por fricción. Mientras el bloquecito cae empuja al bloque grande contra la pared, pero ésta lo aguanta y no le permite movimiento. Al ascender por la otra parte de la semiesfera, empuja al bloque grande hacia adelante y le comunica cantidad de movimiento horizontal, así como energía cinética. Al llegar a su punto de máxima altura, retorna y vuelve a caer, y en esa caída continúa empujando al bloque grande hacia la derecha. Al llegar a la parte más baja en ese retorno comienza a subir por la oquedad y ya empieza a frenar al bloque grande. Por tanto, la máxima velocidad del bloque grande se alcanza cuando el pequeño pasa por el punto más bajo, de retorno hacia la pared. La energía inicial del sistema es E = Ug = m2gr. Al llegar abajo en el primer descenso, la energía se convierte en cinética y la velocidad que alcanza hacia la derecha será v02 = . Su momento lineal será en ese momento p02 = m2v02, y éste será el momento total horizontal del sistema en ese instante; y se conservará en lo sucesivo, pues ya se elimina la acción horizontal externa de la pared. Cuando el bloquecito llega al punto más bajo en su retorno, su rapidez será v2 hacia la pared y la del bloque grande será v1 hacia la derecha. Entonces: m2v02 = m1v1 – m2v2 y ½ m2v022 = ½ m1v12 + ½ m2v22, donde v02 ya se conoce. Resolviendo para v1: v1 =2m2v02/(m1+m2) = 2m2

/ (m1+m2) 33

04.2-En la situación planteada se conservan el momento lineal horizontal del sistema y la energía mecánica total, dado la ausencia de fricción y de cualquier otra fuerza horizontal externa al sistema cuñas-cubito. Durante el descenso por la primera cuña, ésta adquiere movimiento hacia la izquierda, con velocidad V, y el cubito adquiere velocidad v hacia la derecha. Se cumplirá: mgh = ½ mv2 + ½ MV2

y 0 = mv – MV

De estas dos ecuaciones se obtiene: v =

(1)

Cuando el cubito asciende por la segunda cuña, la empuja hacia adelante y cuando llega a su máxima altura, h’, comparte la velocidad horizontal de la cuña, V’, y se cumplirá: mv = (m + M)V’

y

½ mv2 = ½ (m + M) V’2 + mgh’

Eliminando V’ de estas dos ecuaciones: 2gh’= v2/ (1 + m/M) Y teniendo en cuenta (1): h’= h / (1 + m/M)2

4.3-El cilindro entra en contacto simultáneamente con los cubos laterales, con los cuales colisiona centralmente. Las fuerzas de contacto en el choque se producen perpendicularmente a las caras de los cubos (Fig. 4.3S), por lo que cada cubo sale con velocidad u (iguales por la simetría de la situación), sin empujar al cubo central. El cilindro a su vez quedará con una velocidad v’ (hacia adelante o hacia atrás). Estos choques serán elásticos y se conservarán la energía y el momento del sistema: ½ mv2 = ½ mu2 + ½ mu2 + ½ mv’2

y

u

mv = mu cos45 + mu cos45 + mv’ v’

O: v2 = 2u2 + v’2 y v = 2u cos45 + v’

u

Pero cos45 = ecuación:

/2, lo cual modifica la segunda

v2 = 2u2 + v’2 y v = u

Fig. 4.3S

+ v’

Resolviendo estas dos ecuaciones se obtiene v’= 0 y u = ( /2) v. Significa que el cubo central no recibe impacto porque el cilindro queda en reposo, por lo que no se mueve, en tanto los cubos laterales salen con las velocidades calculadas. 34

4.4-En los choques plásticos los cuerpos se pegan, pero pueden quedar pegados y con velocidad hacia alguna parte, o podrían quedar pegados y en reposo. La energía de cada cuerpo es inicialmente potencial, y respecto al fondo de la semiesfera es mgr, para el primero, (4/3)mgr para el segundo y (5/3)mgr para el tercero. Cada una se transforma en cinética en esa misma cantidad cuando llega a la parte de abajo del interior de la semiesfera. Si las tres esferas se sueltan a la vez desde el borde superior, llegan a la vez al fondo con la máxima energía cinética posible, KA + KB + KC = 4mgr, y si se pegaran y quedaran en reposo, toda esa energía se convertiría en calor Q, la máxima cantidad posible si quedan en reposo. Ahora, para que las tres queden pegadas en reposo después del choque no solamente hay que soltarlas simultáneamente desde la misma altura, sino desde posiciones que al llegar abajo tengan una cantidad de movimiento total nula antes del choque, pues sólo así quedarán en reposo, conservando el momento lineal del sistema. Las posiciones iniciales están sugeridas por las relaciones entre las masas, 3:4:5, pues son los números pitagóricos típicos: al llegar abajo todos con la misma rapidez, v = , sus momentos lineales estarán en la relación de sus masas: pA : pB : pC = 3:4:5. Así, habrá que colocarlos en el borde superior de la semiesfera, en tres puntos A, B, C tales que cuando los cuerpos desciendan al centro de la semiesfera, sus momentos lineales pA, pB, pC queden formando un triángulo pitagórico, de ángulos 53o y 37o. La Fig. 4.4S muestra la situación. En las posiciones iniciales, la de masa m debe estar en A, a 90o de la de masa (4/3)m, en B, y a 127o de la de masa (5/3)m en C.

B

Vista superior

(4/3)m

pC pC

m 90 37 53

C

A

pB

(5/3)m

Fig. 4.4S 4.5-La Fig. 4.5S (próxima página) muestra la situación. La bala empuja a la esfera horizontalmente cuando la atraviesa y, despreciando las dimensiones de la esfera en relación con la altura de la mesa, ambas llegan a la vez al piso. La componente horizontal del momento lineal total de la bala y la esfera se conserva pues no hay fuerzas horizontales externas (se desprecia el aire), y la fuerza externa de gravedad altera solamente a la componente vertical del momento lineal, no a la componente horizontal. Ahora bien: el momento lineal total del sistema es igual a la masa total del sistema por la velocidad del centro de masa del sistema. El centro de masas del sistema se mueve horizontalmente con la misma velocidad V antes y después del impacto: (M + m)V = mv + Mvoe donde voe = 0 es la velocidad de la esfera antes del choque. Por tanto: V = mv / (m + M) = 23.8 m/s

35

x xe

O

xcm

xb

Fig. 4.5S Cuando la bala y la esfera llegan al piso, ha transcurrido en la caída un tiempo: t=

= 1.01 s

En ese tiempo el centro de masas avanza horizontalmente: xcm = Vt = 24.0 m Pero el centro de masas de dos cuerpos satisface la relación: xcm = (M xe + mxb) / (M + m) Y sabemos que xe = 20.0 m y xcm = 24.0 m. Entonces: xb = [(M + m) xcm – Mxe] / m = 104 m Para calcular la fracción de energía perdida necesitamos las velocidades de la bala y la esfera al terminar el choque. Tales velocidades horizontales podemos calcularlas por las distancias horizontales que avanza cada una durante el tiempo de caída: v’e = x2/t = 19.8 m/s y v’b = xb/t = 103 m/s. Entonces: ΔK / K = (½mvb2 - ½mv’b2 - ½Mv’e2) / (½mvb2) = 0.93.

4.6-En el instante en que se pone vertical y tirante el hilo, la velocidad de la esfera de masa 2m es: v=

y ha transcurrido un tiempo t0 tal que v = vo – gt0, por lo que:

t0 = (vo – v) /g = Instantáneamente el hilo tirante frena a la esfera de masa 2m y le imprime una nueva velocidad u1, a la vez que levanta a la segunda esfera con una velocidad inicial u2. Este tirón de la cuerda se comporta como un choque elástico entre las bolas (son indeformables las esferas e inextensible el hilo), por lo que: 2 mv = 2 mu1 + mu2

(conservación del momento lineal)

36

Y: ½ (2m) v2 = ½ (2m) u12 + ½ m u22

(conservación de la energía cinética)

Resolviendo el sistema: u1 = v/3 y u2 = 4v/3 La esfera de abajo arranca con más velocidad que la que tiene la primera después del tirón, por lo que la alcanzará y chocara con ella. Después del tirón, ambas esferas continúan hacia arriba influidas solamente por la fuerza de gravedad (el hilo flojo no hace fuerza) y satisfacen: y1 = l + u1 t1 – ½ gt12

y

y2 = u2 t1 – ½ gt12

Cuando choquen: y1 = y2 Entonces: l + u1 t1 – ½ gt12 = u2 t1 – ½ gt12 O sea: t1 = l / (u2 – u1) = l /(4v/3 – v/3) = l / v = l / Así, chocan un tiempo t1 después de haberse producido el tirón del hilo. El tiempo t transcurrido desde que se tiró la primera esfera es: t = t1 + t0 ambos conocidos ya. El punto en que se produce el choque está por arriba del punto de lanzamiento una distancia dada por: y2 = u2 t1 – ½ gt12 = (4v/3)(l/v) – ½ g (l/v)2 = l {4/3 − 1/ [2(vo2/gl - 2)]}

4.7-Durante el descenso del péndulo matemático (péndulo simple), la masa desciende una altura igual a la longitud del péndulo, l, y por conservación de la energía llega al punto inferior con una velocidad horizontal de rapidez v = . Justo al comenzar su ascenso el punto de suspensión se acelera verticalmente, por lo que en el sistema no inercial asociado al punto de suspensión aparece una fuerza de gravedad aparente mayor, dada por m(g+a). En este sistema no inercial, la energía potencial aparente de la masa del péndulo, cuando éste suba una altura h será U = m (g+a)h. Conservando la energía durante el ascenso, y teniendo en cuenta que en el punto inferior de la masa del péndulo la velocidad horizontal es la misma en el sistema fijo a Tierra y en el sistema no inercial acelerado verticalmente hacia arriba, tendremos: ½ mv2 = m (g+a) h Sustituyendo el valor encontrado para v y despejando: h = l g / (g+a) Se ve que el péndulo asciende una altura menor que l.

37

(1)

El máximo ángulo θ que asciende el péndulo se encontrará por trigonometría, teniendo en cuenta que (Fig. 4.7S): o

h = l – l cosθ, que da un ángulo menor que 90 .

θ l

l h

Igualando con la expresión (1) y despejando: cosθ = a/(a + g)

Fig. 4.7S

4.8-Llamemos m a la masa de la nave y Δm, a la masa de gas expulsada. La nave está inicialmente en una órbita circular que satisface: mv2/(R + h) = GmM/(R + h)2 donde M es la masa de la Luna. Simplificando: v2 = GM/(R + h) = gR2/(R + h) donde g = GM/R2 es la aceleración de la gravedad en la superficie lunar. Esta velocidad es la de la nave en la órbita inicial. Evaluándola: v = 1652 m/s Al expulsar gas instantáneamente, en ese mismo punto de la órbita la nave se frena a un nuevo valor de velocidad w con el que realiza una elipse, cuya velocidad es u al pasar rasante a la superficie lunar. En los extremos del eje mayor de la elipse se cumplirá: ½ mw2 – GmM/(R+h) = ½ mu2 – GmM/R

(conservación de energía)

mw(R + h) = muR

(conservación del momento angular)

De estas dos ecuaciones se despeja w: =

= 1628 m/s

Tenemos entonces las velocidades de la nave antes y después de expulsar la masa Δm de gas. Por conservación del momento lineal al expulsar el gas: mv = (m – Δm)w + (Δm)(w + vgn) Despejamos Δm:

donde vgn= 10000 m/s

Δm = m (v – w) / vgn = 2.4 t

38

4.9-No hay fricción y la fuerza normal de contacto no trabaja sobre la partícula, por lo que la única fuerza que trabaja es la de gravedad, que es conservativa; significa que la energía mecánica se conserva en este movimiento. Llamando v a la velocidad en el punto más bajo, cuando la partícula desciende una altura h podemos plantear: ½ mvo2 = ½ mv2 – mgh

O R NR mg

vo

h

r Fig. 4.9S

(1)

Por otra parte, la fuerza normal de contacto apunta siempre hacia el punto O en el centro de la base del hemisferio, en el eje vertical de simetría (Fig. 4.9S), por lo que no produce torque vertical respecto a ese eje (sólo produce torque horizontal). Y el peso apunta hacia abajo, por lo que produce un torque horizontal respecto a O en cada momento, y este torque tampoco tiene componente vertical jamás; significa que la componente vertical del momento angular de la partícula se conserva (Lo,vert = Lf,vert ). En el punto más alto de la trayectoria y en el punto mas bajo, la partícula tiene velocidad horizontal y momento angular solamente vertical, por lo que podremos plantear: mvoR = mvr

(2)

donde R y r se miden perpendicularmente desde el eje vertical a la partícula en cada punto (Fig. 4.9S). De la misma figura se ve que: r= De (2) y (3): v = vo R/r = vo R/

(3) = vo /

Sustituyendo en (1): vo2 = vo2 / [1 – (h/R)2] – 2gh Da una ecuación cúbica en h: h3 + (vo2/2g) h2 – R2h = 0 Una solución es h = 0, que corresponde a la altura inicial. La otra solución sale de la cuadrática: h2 + (vo2/2g) h – R2 = 0 Obteniéndose: h = −(vo2/4g) ± Es válida la solución positiva: h = Si vo = 0 la partícula desciende h = R hasta el fondo del casco antes de volver a subir. Si vo ≠ 0, la partícula desciende una altura h h1, por lo que el caso (a) es el de equilibrio estable, y el (b), el de equilibrio inestable. 50

5.5-Cuando un cuerpo descansa en equilibrio sobre una superficie horizontal, la fuerza normal de contacto que experimenta pasa justamente por el centro de masas del cuerpo. Pero si la superficie se acelera hacia la derecha, la normal se corre hacia la izquierda, como habremos notado cuando estamos en el pasillo de un ómnibus, de frente a las ventanillas, y el ómnibus arranca: automáticamente el pie que queda más atrás en el pasillo es sobre el que se recarga el cuerpo, y la fuerza normal sobre ese pie es mayor que sobre el de alante. N En un cubo, la normal se corre en dirección opuesta a la acelel ración a que se le somete, y se corre más mientras mayor sea la Fi aceleración. Si la normal llega al borde posterior del cubo y aún l Fs la aceleración aumenta más, ya la normal no podrá correrse más O atrás y el cubo se voltea. La Fig. 5.5S muestra la situación mg extrema, observada desde el sistema no inercial que se mueve Fig. 5.5S con aceleración a (junto con el bloque), cuando la normal N está ya sobre el saliente O. La fuerza de inercia Fi = ma (1), y la fuerza Fs del saliente, hacia adelante, crean un torque que tiende a rotar el cubo en contra de las manecillas del reloj. La fuerza de inercia queda aplicada al centro de masas del bloque.

La normal y el peso mg crean un torque recuperador a favor de las manecillas del reloj que se opone a que el cubo se voltee. En el sistema no inercial ambos torques se contrarrestan mientras el cubo no se voltee. Llamando l al lado del cubo tendremos, cuando el cubo está ya a punto de voltearse: Fil/2 – mgl/2 = 0, por lo que Fi = mg. Comparando con (1): a = g. Ésta es la aceleración que debe adquirir como mínimo el bloque para que el cubo esté a punto de voltearse. Ahora, la fuerza F que hay que aplicar al bloque de masa M debe arrastrar al bloque y al cubo, por lo que debe valer: F = (m + M) g.

5.6-La velocidad del sistema varilla-bloque después del choque es la velocidad de su centro de masas vc. Como en el sistema se compensan las fuerzas externas (pesos y normales de contacto), el momento lineal del sistema se conserva y la velocidad del centro de masas no varía con el choque. Por conservación del momento lineal: Mv = 2Mvc. De aquí: vc = v/2, que es la velocidad final del sistema. Por otra parte, los torques externos también se anulan, por lo que se conserva el momento angular del sistema respecto a cualquier punto, o eje, de un sistema inercial: Lantes = Ldespués, y de esta condición podremos calcular la velocidad de rotación, ω, después del choque. Tanto el piso sobre el que se desliza el sistema varilla-bloque, como un sistema ligado al centro de masas del conjunto, sirven como sistemas inerciales de referencia. Aquí tomaremos un eje de momentos fijo al piso (en el próximo problema se escogerá un sistema ligado al centro de masas), que entre en el 51

piso por el punto en que el centro de masas del conjunto se encuentre en el momento del impacto del bloque y la varilla, punto que estará a una distancia l/4 del extremo de la varilla y del centro de la misma (Fig. 5.6S). Respecto a este punto fijo, solamente el bloquecito tiene momento angular antes del choque, por lo que Lantes = Mvl/4; y después del choque, todo el sistema rota unido como un solo cuerpo, con momento angular de espín, Lespín = Iω y sin momento angular orbital del centro de masas, puesto que dicho centro se mueve sobre la recta que pasa por el eje escogido (Lorb = 0). El momento de inercia del sistema respecto al eje escogido es la suma de los momentos de inercia de cada parte: parte:

Vista superior

M Centro varilla

l/4

Centro masas

v/2

l/4 M

v

Fig. 5.6S

I = Ivarilla + Ibloque = [Ml2/12 + M (l/4)2] + M (l/4)2 = (5/24)Ml2 donde se tuvo en cuenta el teorema de los ejes paralelos para la varilla. Entonces, por conservación del momento angular: Mvl/4 = (5/24) Ml2ω Y despejando: ω = (6/5) v/l

5.7-Las fuerzas sobre las dos varillas se compensan (el peso de cada una con la normal de contacto), y no introducen torques externos. Por tanto, se conservan el momento lineal y el momento angular del sistema de varillas. La conservación del momento lineal nos lleva a plantear: mv = 2mvc, donde vc es la velocidad del centro de masas del conjunto, que vale lo mismo antes que después del choque. Entonces: vc = v/2, y representa la velocidad del conjunto después del choque. Para calcular la velocidad de rotación deberemos tener en cuenta la conservación del momento angular del conjunto. La Fig. 5.7S muestra la rotación de las varillas después de pegarse en el choque: rotan alrededor del centro de masa del conjunto, a favor de las manecillas del reloj. Para los cálculos, tomemos el centro de masas del conjunto como sistema inercial de referencia (todo el tiempo se mueve con velocidad rectilínea v/2). 52

Vista superior

l v

m l/4

ω

CM

l/4 m l

Fig. 5.7S

El momento de un sólido respecto a un punto es la suma vectorial de su momento angular orbital, Lorb=mVr senθ y su momento angular de espín, Lspin= Iω; en nuestro caso se reduce a la suma algebraica pues ambos apuntan perpendicularmente al plano del papel. Antes del choque, respecto al centro de masa móvil del conjunto, ambas varillas tienen momento angular orbital, y ninguna lo tiene de espín, pues ninguna rota; la rapidez de ambas respecto al centro de masa es v/2, en direcciones opuestas, pero ambas con momento angular a favor de las manecillas; el brazo de momento de cada una es rsenθ = l/4, por lo que las dos juntas tienen momento angular orbital antes del choque: Lorb = m(v/2) l/4 + m(v/2) l/4 = mvl/4. Después del choque, las dos varillas juntas forman un solo cuerpo que rota alrededor de su centro de masas común, por lo que su momento angular se expresará como Lespin = Iω, donde ω es la velocidad angular del conjunto y la I representa el momento de inercia de las dos varillas respecto al centro de masas común. Para cada varilla, este momento puede calcularse por el teorema de los ejes paralelos, puesto que el eje de rotación está desplazado l/4 del eje central de cada varilla: I1 = ml2/12 + m(l/4)2 = (7/48)ml2. Las dos varillas juntas aportan un momento de inercia total de I = 2I1 = (7/24)ml2. Entonces, después del choque: Lespín = (7/24)mωl2. Y por conservación del momento angular: mvl/4 = (7/24)mωl2. Despejando: ω = (6/7)v/l La energía perdida es la diferencia de las energías cinéticas antes y después del choque: ΔE = ½ mv2 – ½ Mvc’2 – ½ Iω2 donde M = 2m, I =(7/24)ml2 ) y vc = v/2. Sustituyendo: ΔE = (1/7)mv2

5.8-La velocidad angular ω que adquiere el disco chiquito es la de rotación alrededor del eje vertical fijo del disco grande cuando finalmente quede en reposo sobre el grande. Ésta será también la velocidad angular final ω del disco grande. La fuerza de fricción entre los discos es interna del sistema compuesto por dichos discos, por lo que no varía el momento angular del sistema. La fuerza de gravedad sobre el disco pequeño introduce un torque externo respecto al eje vertical central del disco grande, pero tal torque se compensa con el que produce la fuerza de reacción que ejerce el eje sobre el disco, impidiendo que se voltee el sistema. En resumen, la resultante de torques externos sobre el sistema de los dos discos es nula, por lo que el momento angular vertical del sistema se conserva respecto al eje central. Entonces, teniendo en cuenta el teorema de los ejes paralelos: 53

½ MR2Ω = ½ MR2ω + (½ mr2 + md2) ω Despejando: ω = Ω / [1 + (m/M) (r2/R2 + 2d2/R2)]

5.9-En (a) la esfera tiene energía cinética de traslación pura: Ko = ½ mvo2. En (c) la esfera tiene energías cinéticas de traslación y de rotación, y puesto que se mueve en rodadura pura se cumplirá además que la velocidad con que avanza, v, y la de rotación, ω, se relacionan por la ecuación v = ωR, donde R es el radio de la esfera. Así, la energía final es: K = ½ mv2 + ½ Iω2 = ½ mv2 + ½ (2/5)mR2(v/R)2 = (7/10)mv2

(1)

Y debemos calcular la fracción: (Ko – K)/Ko. La fricción cinética que surge es fk = μkN = μkmg. De aquí: ma = μkmg, por lo que la aceleración de frenado es a = μkg. La propia fricción produce un torque alrededor del eje central de la esfera que produce la aceleración angular que la hace rotar: η = fkR = Iα. O sea: μkmgR = (2/5)mR2α, por lo que μkg = (2/5)αR y se tendrá: α = (5/2) μkg/R. La aceleración a refleja un frenado en la traslación de la esfera (disminuye v) y la aceleración α refleja un incremento de la velocidad angular de rotación (aumenta ω). En este proceso de aumento de ω (a partir de cero) y disminución de v (a partir de vo) llegará un momento en que se cumplirá v = ωR, y de ahí en adelante se producirá una rodadura pura (como en la Fig. 5.9-c). En el tiempo t que dura este proceso la esfera alcanza una velocidad de traslación: v = vo – at = vo – μkg t

(2)

Y en ese mismo tiempo la esfera alcanza una velocidad de rotación: ω = 0 + αt = (5/2) μkg t/R De aquí: ωR = (5/2) μkg t Pero al final de este tiempo t se establece la rodadura pura, por lo que v = ωR y quedará: v = (5/2) μkg t. De aquí se obtiene que: μkg t = (2/5)v. Sustituyendo en (2): v = vo – (2/5)v. Despejando: v = (5/7)vo . Sustituyendo en (1): K = (7/10) m (25/49) vo2 = (5/14)mvo2 Finalmente: ΔK / Ko = [½mvo2 – (5/14)mvo2] / ( ½ mvo2) = 2/7 54

5.10-Sean m y R la masa y el radio del cilindro central. Los laterales, siendo de la misma densidad, tendrán masas: ml = ρV = ρlπ4R2 = 4m. El momento de inercia del cilindro central será ½mR2, y el de cada cilindro lateral será ½ml (2R)2 = 8mR2. El momento de inercia total del cuerpo respecto al eje central será Io = (33/2)mR2 y la masa total del cuerpo será M = 9m. En el descenso por el plano inclinado se conserva la energía mecánica (es una rodadura pura), por lo que: Mgh = ½MV2 + ½Ioω2, donde V = ωR. Esta V es la velocidad del centro de masas del cuerpo en el borde final del plano inclinado, justo cuando va a pasar al plano horizontal. Desarrollando: (9m)gh = ½(9m)V2 + ½(33/2)mR2(V/R)2 = (51/4)mV2 De aquí: V = La velocidad angular en ese instante es: ω = V/R. Y el borde de los cilindros laterales que hace contacto con el piso horizontal en ese instante gira con una velocidad de traslación V’= 2ωR = 2V, por lo que va hacia atrás el doble de rápido que lo que el centro de masas avanza en la horizontal. Esto provoca que haya deslizamiento al pasar al piso horizontal, con una fricción cinética dirigida hacia adelante (la rueda desliza hacia atrás). Esta fricción hacia adelante hará aumentar la velocidad de traslación del cuerpo, a la vez que produce un torque opuesto a la rotación de la rueda que hará disminuir su velocidad angular. De este modo, llegará un momento en que el valor ωf (2R) = vf, y de ahí en adelante el cuerpo seguirá en rodadura pura. Así la aceleración de impulsión en la traslación será a = f /M = f / (9m) y la impulsión durará un tiempo tal que: vf = V + at = V + f t /(9m) , por lo que t = (vf − V)9m/f. Durante ese tiempo actúa un torque de frenado que cambia la velocidad angular de ω = V/R a ωf = vf /2R, con una aceleración de frenado: α = η/Io = fR/ [(33/2)mR2]. Se cumple: ωf = ω – αt Sustituyendo todos los valores: vf /(2R) =V /R – {fR/ [(33/2)mR2]}(vf − V)9m/f Simplificando: vf = 2V − (36/33)vf + (36/33)V De aquí: vf = (34/23)V = (34/23)

≈ 1.2

5.11-Si no hay fricción no existirán fuerzas horizontales sobre el sistema, por lo que el centro de masas del sistema caerá verticalmente. La suma de las tres energías cinéticas de las tres esferitas será igual a la energía potencial inicial del sistema: 55

½ mv12 + ½ mv22 + ½ mv32 =mgl/2 + mgl

(1)

donde consideramos “1” la esfera inferior y “2”, la del centro. En el instante en que la esfera superior llega al piso, su velocidad v3 es vertical hacia abajo, al igual que v2 de la esfera central, pero la velocidad de la esfera “1” es cero: v1 = 0, pues si tuviera aún velocidad horizontal hacia la izquierda, arrastraría al sistema rígido hacia la izquierda, lo cual es imposible. Entonces, respecto a la esfera “1”, las velocidades v2 y v3 son tales que v3 = 2v2, pues en el instante final las otras dos esferas se ven describiendo circunferencias hacia abajo en torno a “1”. Y esta relación de velocidades es la misma respecto al piso, pues la bola “1” está en reposo en el instante final. Entonces, la ecuación (1) queda: 0 + ½ m(v3/2)2 + ½ mv32 =mgl/2 + mgl Y despejando: v3 = 2 Si la esfera inferior estuviera fija al gozne, el sistema caería rotando todo el tiempo alrededor de esa esfera inferior, y el resultado sería el mismo de antes.

56

6-Hidrostática. Problemas.

6.1-Un cilindro de finas paredes lleno de gas, de masa m, altura h y área de la base S, flota en agua como muestra la Fig. 6.1. Como resultado de la pérdida de hermeticidad en la parte inferior del cilindro, la profundidad de inmersión aumenta un Δh. Determine la presión inicial p1 del gas en el cilindro. La presión atmosférica es igual a po y la temperatura no cambia.

h S

Fig.6.1

6.2-Se vierte líquido de densidad ρ en un recipiente cilíndrico vertical, el área de cuyo fondo es S. ¿En cuánto varía el nivel del líquido en el recipiente si en él se introduce un cuerpo de masa m de forma arbitraria, con heterogeneidades y huecos en su interior, que no se hunde?

6.3-En el fondo de un recipiente cilíndrico chato de área S1 y altura h abrieron un agujero de área S2 y colocaron en él un tubo de plástico. La masa del recipiente con el tubo es m. El recipiente se para sobre una placa de goma dura, con su fondo hacia arriba, como un embudo invertido (Fig. 6.3). Por el tubo vierten agua. ¿Hasta qué altura puede subir el agua por el tubo sin que se salga? No tenga en cuenta la tensión superficial.

S2 S1

h

Fig.6.3

6.4-Una pecera tiene agua (densidad: 1.00 g/cm3) hasta una altura de 50.0 cm. ¿Qué fuerza ejerce el agua sobre una pared de la pecera de 80.0cm de ancho? ¿Qué fuerza neta sufre la pared hacia afuera de la pecera? (Presión atmosférica: 1.01×105 Pa)

6.5-En una bolsita ligera de hilo hay 100 monedas de dimensiones idénticas, pero unas de oro y otras de plata. El peso de la bolsita con las monedas es de 127.39 g en el aire, pero sumergida en agua tiene un peso aparente de 119.54 g. Cuántas monedas hay de cada una, sabiendo que la densidad del oro es 19.3 g/cm3 y la de la plata, 10.5 g/cm3.

57

6-Hidrostática. Soluciones.

6.1-Sea p2 la presión del gas en el interior del cilindro cuando éste se encuentra en el estado final de equilibrio. El cilindro va a estar sometido a fuerzas de presiones por su interior y por su exterior, más la fuerza de gravedad. Entre las fuerzas por su exterior están las que lateralmente ejercen la atmósfera y el agua, pero éstas se compensan por aparecer en puntos opuestos del cilindro, al mismo nivel. Por debajo, el agua ejerce fuerza de presión hacia arriba, pero se compensa con la que hace hacia abajo el agua en el interior del cilindro, a la misma profundidad h respecto a la superficie libre del agua. Las fuerzas de presión por el interior, sobre las paredes, también se compensan por actuar sobre puntos opuestos. Quedan solamente sin compensarse la fuerza de presión que hace la atmósfera hacia abajo por la cara superior y la fuerza de presión que hace el gas interior sobre la misma cara, hacia arriba. Pero la diferencia entre estas dos fuerzas debe compensarse con la fuerza de gravedad sobre el cilindro. Luego: p2S = poS + mg. De aquí: p2 = po + mg/S

(1)

Y por la ley de Boyle: p2 (h – Δh) S = p1S h

(2)

De (1) y (2): p1 = (po + mg/S) (1 – Δh / h)

6.2-Si el cuerpo introducido flota, la fuerza de empuje será igual al peso del cuerpo introducido. Por Arquímedes, esta fuerza es igual al peso del volumen de fluido desplazado por el cuerpo. Si Δh es la altura del líquido desplazado: mg = g (ρSΔh) Luego: Δh = m / (ρS)

6.3-En la medida que se echa agua y sube el nivel por el tubo estrecho aumenta la presión hacia arriba que ejerce el agua sobre la superficie horizontal del recipiente, de área S1 – S2. La fuerza de presión hacia arriba va igualmente aumentando, hasta que levantará el recipiente cuando sobrepase la fuerza de gravedad sobre el cilindro. La situación límite ocurre cuando: ρg (H – h)(S1 – S2) = mg donde H es la altura total que asciende el liquido sobre la superficie de apoyo del recipiente. De aquí: H = h + m/[ρ(S1 – S2)] 58

6.4-a)La fuerza que ejerce el agua está dada por la presión del agua sobre la pared. Esta presión crece linealmente con la profundidad, por lo que la presión media sobre el área de la pared será el valor de la presión a la mitad de la altura del agua, h = 25.0 cm = 0.250 m: p = ρgh + po = 1.00×103 ×9.80×0.250 + 1.01×105 = 1.03×105 Pa La fuerza es entonces: F = pA = 1.03×105 ×0.500×0.800 = 4.14×104 N b)Del lado de afuera, la presión atmosférica empuja hacia adentro al vidrio, por lo que la fuerza neta del agua hacia afuera es: Fneta= ρghA = 1.00×103 ×9.80×0.250×0.500×0.800 = 980 N Una fuerza igual y opuesta estarán haciendo los soportes de la pared para mantenerla en equilibrio.

6.5- Sean: WA = 127.39 gf el peso total de las monedas en el aire, WW = 119.54 gf su peso total en el agua, wo el peso de cada moneda de oro, wp el peso de cada moneda de plata, ρo, ρp, sus respectivas densidades y ρ, la densidad del agua. Las cantidades de cada tipo de moneda serán No y Np. Entonces: WA = Nowo + Npwp

(1)

y el peso aparente será el peso real menos la fuerza boyante : WW = (Nowo + Npwp) – (Nowoρ/ρo + Npwp ρ/ρp)

(2)

De (1): Nowo = WA − Npwp. Sustituyendo en (2) y arreglando : WW + WA (ρ/ρo – 1) = Npwp (ρ/ρo − ρ/ρp) Despejando y evaluando: Npwp = 28.77 gf. Y sustituyendo en (1): Nowo = 98.62 gf. Dividiendo miembo a miembro estas dos igualdades: Npwp / Nowo = 0.2917

(3)

Pero como las monedas son de volúmenes iguales: Npwp / Nowo = Npρp / Noρo = 0.5440 Np / No. Sustituyendo en (3) y arreglando: Np / Na = 0.5362. Y por regla de las proporciones: (Np + No)/ No = (0.5362 + 1) / 1, donde Np + No = 100. O sea: 100/No = 1.5362. De aquí No = 65, y en consecuencia: Np = 100 – 65 = 35.

59

7A-Termodinámica I. Problemas.

7.1-Dos cuerpos con masas m1 y m2 se mueven al encuentro uno del otro, en direcciones mutuamente perpendiculares, con velocidades v1 y v2. Como resultado del choque los cuerpos se pegan. Determine la cantidad de calor Q liberada como resultado del choque.

7.2-Dos esferas iguales están a la misma temperatura. Una de las esferas se encuentra sobre una superficie horizontal aislante y la otra pende de un hilo aíslante. A ambas se les comunican iguales cantidades de calor, tan rápidamente que no hay pérdidas de calor al ambiente. ¿Serán exactamente iguales, o serán diferentes las temperaturas de las esferas después de calentarlas?

7.3-En un recipiente térmicamente aislado se encuentran dos líquidos con temperaturas iniciales T1 y T2, y calores específicos c1 y c2, separados por un tabique no conductor. El tabique es retirado y después del establecimiento del equilibrio térmico la diferencia entre la temperatura inicial del líquido 1 y la temperatura que se establece en el recipiente es la mitad de la diferencia entre las temperaturas iniciales de los líquidos. Encuentre la razón de las masas de los líquidos, m1 / m2 .

7.4-En un calorímetro aislante se vierten m1 = 0.50 kg de agua a temperatura T1 =+15ºC. En el agua se coloca un pedazo de hielo de masa m2 = 0.50 kg a temperatura T2 =−10ºC. Encuentre la temperatura de la mezcla cuando se alcanza el equilibrio térmico. Calores específicos del agua y del hielo: c1 = 4.2×103 J/kg.K, c2 = 2.1 ×103 J/kg.K. Calor de fusión del hielo: L = 3.3×105 J/kg.

7.5-Un calorímetro de cobre de masa m1, con una masa de agua m2, está a la temperatura T1. En el calorímetro se echa hielo de masa m3 y temperatura T2. (a) Determine la temperatura y las masas del agua y del hielo después de establecerse el equilibrio térmico, para valores cualesquiera de m1, m2, m3, T1 y T2. Escriba la ecuación del balance térmico del sistema. (b) Determine la temperatura y las masas de agua y hielo en estado de equilibrio térmico si m1 = 1.0 kg, m2 = 1.0 kg, m3 = 2.0 kg, T1 = 10oC y T2 = −20oC. Despreciar las pérdidas de calor al ambiente. Considere la presión atmosférica como la normal. Calores específicos del cobre, del agua y del hielo: c1 = 0.39 kJ/kg oC, c2 = 4.2 kJ/kg oC, c3 = 2.1 kJ/kg oC. Calor de fusión del hielo: L = 330 kJ/kg.

60

7.6-En un recipiente hay un volumen V1 = 3.0×10-4 m3 de tolueno a una temperatura t1 = 0ºC y en otro, un volumen V2 = 1.1×10-4 m3 de tolueno también, a una temperatura t2 = 100ºC. ¿Qué volumen V ocupa el tolueno después de mezclarlos, si su coeficiente de dilatación volumétrica es α = 0.0001 K-1? Despreciar las pérdidas de calor.

7.7-Dos cilindros verticales tienen secciones transversales diferentes y están conectados por sus bases mediante un fino tubo. Las partes superiores de cada cilindro están cerradas por sendos pistones, de masas m1 = 1.0 kg y m2 = 2.0 kg. Por la parte superior de los pistones hay vacío total, y por la parte inferior hay un gas ideal a temperatura constante. Los pistones están situados a alturas iguales, ho = 0.20 m. ¿Cuál será la diferencia de alturas de los pistones si se aumenta la masa del primer pistón hasta la masa del segundo?

7.8-Un fino tubo en forma de U, cerrado por uno de sus extremos, consiste en tres tramos de longitud l = 250 mm cada uno, doblados en ángulos rectos. Las partes verticales del tubo están llenas de mercurio hasta la mitad (Fig.7.8). Lentamente se calienta el gas de la parte cerrada del tubo, de tal modo que expulsa del tubo todo el mercurio. Determine qué trabajo W realiza el gas en el tubo para expulsar totalmente el mercurio. La presión atmosférica es po=1.0×105 Pa, la densidad del mercurio es ρ = 13.6×103 kg/m3 y la sección transversal del tubo es S = 1.0 cm2.

Fig.7.8

7.9- Un intercambiador térmico de longitud l consiste en un tubo de sección transversal de área 2S por dentro del cuál pasa otro tubo de sección transversal de área S (Fig.7.9). El tubo interior T1i

T1f

Tubo 1 v T2f

S

v

2S v ( l >>

l

T2i Tubo 2

)

Fig.7.9

es de paredes finas y conductoras. Toda la construcción está aislada térmicamente del medio exterior. Por ambos tubos circula líquido de densidad ρ y calor específico c, y en ambos tubos la 61

rapidez con que circula el líquido es v, pero moviéndose en direcciones opuestas. Las temperaturas del líquido en las entradas de cada tubo al intercambiador son T1i (ambiente) y T2i (caliente). Determine las temperaturas T1f y T2f del líquido en las salidas del intercambiador. Considere que la cantidad de calor transferido en cada punto, por unidad de tiempo a través de la unidad de área, es proporcional a la diferencia de temperaturas, con un coeficiente de proporcionalidad conocido, k. La conducción térmica del propio líquido a lo largo del flujo es despreciable.

7.10-La Fig.7.10 muestra un calorímetro de flujo continuo. Consiste en un tubo de vidrio sobre dos soportes aislantes, Z, en cuyo interior hay una resistencia eléctrica de calentamiento, R, que se extiende a lo largo del tubo, acoplada a un circuito externo a través de dos tapones aislantes en los extremos del tubo. Se introducen dos termómetros, T1 y T2, cerca de la entrada y de la salida del flujo de líquido en el tubo. Por A entra el líquido cuyo calor específico se desea medir, y sale por B hacia un tanque M Se usa un mecanismo (“frasco de Mariotte”) que asegura un flujo constante de líquido todo el tiempo que dure la medición. T1

T1 A

R B Z

H

Z

M Fig.7.10

En un experimento de medición se trabaja con un líquido cuya densidad es 0.88 g/cm 3. Se establece el flujo de líquido por el tubo y se conecta la resistencia eléctrica. Durante un tiempo cambian las temperaturas de los termómetros, hasta que a los pocos minutos se estabilizan, registrando el termómetro T1 una temperatura estable de 40.0oC y el T2, una temperatura de 32.5oC. En ese momento se introduce un recipiente H a la salida del tubo para recoger líquido por 60.0 s, durante los cuales salen 475 cm3 de líquido. La resistencia eléctrica estuvo conectada durante ese tiempo a una fuente eléctrica alterna de 120 V (voltaje eficaz) que proveía una corriente de 2.20 A (corriente eficaz). a)Calcule con estos datos el calor específico del líquido.

62

b)Explique cómo afectarían al resultado la absorciones de calor por parte de todos los componentes del calorímetro. c)Explique bajo qué rango de valores debe establecer las mediciones para minimizar las pérdidas de calor al ambiente

7.11-Un largo tubo vertical de ladrillos está lleno de hierro colado (Fig. 7.11). El extremo inferior del tubo se mantiene a una temperatura T1>Tfusión, y el extremo superior a T2T2. Entonces, el calor perdido por el líquido más caliente es igual al ganado por el líquido más frío: m1c1 (T1 – T) = m2c2 (T – T2) De aquí: m1/m2 = [c2 (T – T2)] / [c1 (T1 – T)] Pero por dato: (T1 – T) = ½ (T1 – T2). Y de aquí: (T1 – T) = (T – T2) Entonces: m1/m2 = c2/c1

7.4-Las masas de agua, m1, y hielo, m2, son iguales; el agua tiene doble calor específico que el hielo, y la diferencia de temperaturas con el 0oC es mayor para el agua. Significa que para llegar a cero grados centígrados el agua desprenderá más calor que el que necesita absorber el hielo para llegar al cero. En definitiva, el exceso de calor que puede entregar el agua alcanza para fundir hielo. Lo que tendremos que averiguar es si se funde sólo una parte (quedando el sistema a cero grados), o si se funde todo (quedando el sistema justamente a cero grados), o si se funde 64

todo y aún pudiera subir más la temperatura de la mezcla (quedando el sistema por arriba de cero grados). El agua, al enfriarse hasta 0oC, desprendería: Q1 = m1c1(0 – T1) = -3.15×104 J El hielo, al calentarse hasta 0oC, absorbería: Q2= m2c2(0 – T2) = 1.05×104 J La diferencia es: ΔQ=│Q1│− Q2 = 2.10×104J, que alcanza para fundir una masa de hielo: Δm = ΔQ/L = 0.064 kg, que es menor que la masa existente de hielo. En resumen, el sistema se equilibra a los 0oC, con un aumento de la fase líquida de 0.064 kg.

7.5-a)Como hay que resolver el problema de manera general, habrá que considerar tres posibilidades: i) cuando toda el agua quede líquida, sobre 0oC, ii) cuando toda el agua quede sólida, por debajo de 0oC, iii) cuando quede parte sólida y parte líquida, a 0oC. En el primer caso, la temperatura final T será T>0, y la ecuación de balance térmico será: m3c3(0 –T2) + m3L + m3c2(T – 0) = (m1c1 + m2c2)(T1 – T), y de aquí se despeja T. En el segundo caso, la temperatura final T será Tm2, todo termina en fase sólida y la temperatura T debe calcularse por el segundo caso (ii). b)Los datos numéricos de este problema conducen al caso (iii) y se obtiene Δm = -0.12 kg, lo que significa que 0.12 kg de agua líquida se transforman en hielo y el sistema queda en equilibrio térmico a 0oC.

65

7.6-Sean V01 y V02 los volúmenes de tolueno a 0oC. Entonces, sus volúmenes a las temperaturas T1 y T2 son V1= V01(1+αT1) y V2= V02(1+αT2). Las masas de uno y otro volumen serán m1=ρoV01 y m2=ρoV02, donde ρo es la densidad del tolueno a cero grados centígrados. Al unirse las dos muestras líquidas se establece un intercambio de calor hasta alcanzar una temperatura de equilibrio T tal que: m1c1 (T – T1) = m2c2 (T2 – T), donde c1 = c2 = ctolueno. Despejando T, y escribiendo las masas en función de los volúmenes a cero grados, se obtiene: T = (V01T1 – V02T2) / (V01 + V02) El volumen de la mezcla a la temperatura final será V = (V01 + V02) (1 + αT). Sustituyendo T y reordenando se obtiene: V = V01(1+αT1) + V02(1+αT2) = V1 +V2 = 4.1×10-4 m3 Significa que lo que se contrae una masa compensa justamente lo que se expande la otra y la final el volumen sigue siendo el mismo del inicio.

7.7-Al inicio, dado el equilibrio existente, las presiones en ambos cilindros son iguales, y sufícientes para sostener el peso de cada pistón. Cuando se coloca más masa sobre el pistón menor presionará más sobre el gas de ese cilindro y se romperá el equilibrio: fluirá gas del cilindro de mayor presión al de menor presión. En este último, la presión no puede aumentar porque el peso del pistón sigue siendo el mismo. En consecuencia, fluirá gas hasta que el pistón menor choque con el borde inferior del cilindro donde surgirá la fuerza normal de contacto necesaria para detener el pistón. Todo el gas del primer cilindro pasará al segundo sin variar la presión que había inicialmente, ni la temperatura de acuerdo con los datos. Por tanto el volumen total sigue siendo el mismo al final: S1ho + S2ho = S2h donde h es la altura final del segundo pistón. Luego: h = ho( 1 + S1/S2) El cociente S1/S2 puede calcularse a partir de la condición inicial de equilibrio de presiones: m1g/S1 = m2g/S2, por lo que: S1/S2 = m1/m2. Entonces: h = ho( 1 + m1/m2) = 0.30 m

7.8-El trabajo total que realiza el gas puede descomponerse en la suma del trabajo que realiza contra la presión atmosférica, más el trabajo contra la fuerza de gravedad al elevar el centro de masas del sistema líquido desde una altura inicial hasta la altura l en que se encuentra el extremo abierto por donde se derrama el líquido. 66

El trabajo de expansión contra la atmósfera es: W1 = poS(2l + l/2) = (5/2)poSl El centro de masa del líquido está inicialmente a una altura h sobre la parte inferior dada por: h = (1/M) [(M/4)(l/4) + (M/4)(l/4) + (M/2)(0)] = l / 8 donde M representa la masa total del mercurio: M = ρ(2Sl). La altura final a la que la presión del gas levanta todo el líquido es l, por lo que el incremento de altura será: Δh = (7/8) l. El trabajo realizado contra la gravedad es igual al aumento de la energía potencial gravitatoria de la masa liquida: W2 = ΔU = MgΔh = (7/4) ρgSl2. El trabajo total es entonces: W = W1 + W2 = (5/2) poSl + (7/4) ρgSl2 = 7.7 J.

7.9- Los flujos de líquido avanzan por secciones de áreas iguales, S para el tubo 2 y (2S – S) = S para el tubo 1. T T2i T2f ΔT= T2i −T1f T1f T1i x Fig.7.9S

Como la velocidad de los líquidos es la misma, y sus áreas de avance son las mismas, sus gastos de masa son iguales. Por cada sección del intercambiador avanzan flujos líquidos iguales y opuestos, por lo que al intercambiar calor, la misma variación de disminución de temperatura que sufre el líquido en el tubo 2 la experimenta como un aumento el líquido en el tubo 1 (en cada punto, o sección del sistema de tubos).

Esto significa que en cada punto a lo largo de los tubos hay la misma diferencia de temperaturas y se producen iguales intercambios de calor. La distribución de temperaturas será como muestra la Fig. 7.9S. En cada punto del intercambiador, la diferencia de temperaturas entre el líquido frío y el caliente será: ΔT= T2i −T1f = T2f −T1i

(1)

El calor cedido lo largo del tubo 2 en un tiempo t será: Q2 = kAintercambio t (T2i −T1f) donde Aintercambio =2πrl, con r = (S/π)1/2, por lo que: Aintercambio = 2πl(S/π)1/2 = 2l(S π)1/2, y: Q2 = k2l (S π)1/2 t (T2i −T1f) 67

(2)

Por otra parte, el calor ganado por el líquido del tubo 1 se puede expresar: Q1 = mcΔT = ρVc (T1f −T1i), donde V = Svt es el volumen de líquido que pasa por el tubo en el tiempo t. Así: Q1 = ρSvtc (T1f −T1i)

(3)

Pero (2) y (3) representan la misma cantidad de calor: k2l (S π)1/2 t (T2i −T1f) = ρSvtc (T1f −T1i) Despejando y hacienda arreglos: T1f = T2i − (T2i −T1i)[ 1 + 2(π/S)1/2 lk/(ρvc)]-1 Sustituyendo en (1) y despejando: T2f = T1i + (T2i −T1i)[ 1 + 2(π/S)1/2 lk/(ρvc)]-1

7.10-(a) La masa que entra por segundo, Δm/Δt, recorre el tubo alrededor de la resistencia y se va calentando. El calor generado por la resistencia en cada segundo, ΔQ/Δt, equivale a su potencia eléctrica, P = IeVe (donde Ie y Ve son los valores eficaces de corriente y voltaje para la corriente alterna). Este calor al inicio calienta a la propia resistencia, al líquido que fluye y al líquido termométrico. Para los tapones y el vidrio el calentamiento es pequeño pues son muy buenos aislantes y demoran mucho en calentarse. Cuando los termómetros no varían más y se estabiliza la diferencia de temperaturas entre los extremos de la resistencia, en cada punto del tubo de flujo ya no hay diferencia de temperaturas entre el líquido y la pared interior del tubo, por lo que el flujo de calor radial al exterior es mínimo y todo el calor generado en la resistencia se consume en calentar el líquido de un extremo al otro. Si pasa una masa Δm desde un extremo al otro de la resistencia, en un tiempo Δt, sufrirá un aumento de temperatura ΔT que se registra con los termómetros ya estabilizados, y el calor absorbido será: ΔQ = (Δm)cΔT. Por unidad de tiempo será: ΔQ/Δt = (Δm/Δt) cΔT. Sustituyendo por la potencia eléctrica: IeVe = (Δm/Δt) cΔT. Pero Δm = ρΔV, por lo que: IeVe = (ρΔV/Δt) cΔT. Despejando: c = IeVe / [(ρΔV/Δt) ΔT] donde: Ie = 2.20 A, Ve = 120 V, ρ = 0.88×103 kg/m3, ΔV = 475×10-6 m3, Δt = 60 s y ΔT =47.0 – 32.5 = 15.5oC. Sustituyendo valores: c = 2.4 kJ/kg.oC

68

b)Cuando se estabiliza la diferencia de temperaturas entre los termómetros, los componentes del calorímetro prácticamente no absorben del calor generado en un breve intervalo de tiempo, como es un minuto (60.0 s). El calor que inicialmente absorben, durante lo cual la diferencia de temperaturas entre los termómetros crece, no entra en los cálculos. c)Para que la aproximación anterior en (b) sea los más aceptable posible será necesario no establecer diferencias de temperaturas muy altas entre el tubo y el medio ambiente. Con unos pocos grados de diferencia de temperaturas casi no se transporta calor del tubo al ambiente (ni por conducción, ni por convección, ni por radiación). Pueden mejorarse los resultados haciendo las mediciones para varios voltajes durante el mismo tiempo y con el mismo flujo, haciendo un gráfico de P contra ΔT, y determinando la pendiente de la mejor recta que surge, que sería igual a (ρΔV/Δt)c, de donde se podría calcular c.

7.11-La corriente de calor hacia arriba por el hierro es la misma en todas partes del tubo en el estado estacionario. Esta corriente de calor satisface la ley de conductividad térmica: ΔQ/(SΔT) = KΔT/Δl Entonces: Kliq (T1 – Tfus) /l1 = Ksol (Tfus – T2) /l2 Por datos: Kliq = kKsol, por lo que k (T1 – Tfus) /l1 = (Tfus – T2) /l2 Despejando: l1 / l2 = k (T1 – Tfus) / (Tfus – T2) Usando las propiedades de las proporciones: 11/ (l1 + l2) = k (T1 – Tfus) / [k (T1 – Tfus) + (Tfus – T2)] que da la fracción solicitada.

69

8A-Física Molecular. Problemas.

8.1-Las paredes de un recipiente, en el cual hay gas a temperatura absoluta T, poseen una temperatura Tp. ¿En qué caso la presión del gas sobre las paredes del recipiente es mayor: cuando las paredes del recipiente están más frías que el gas ( Tp< T), o cuando están más calientes ( Tp> T)?

8.2-Un recipiente de capacidad V = 30 litros está dividido en tres partes iguales por delgados tabiques semipermeables (Fig. 8.2). En la parte izquierda se introdujo mH = 30 g de hidrógeno,

H2

O2

N2

Fig.8.2

en el medio, mO = 160 g de oxígeno y en la derecha, mN = 70 g de nitrógeno. A través del tabique izquierdo solamente puede difundirse el hidrógeno, y a través del derecho, el hidrógeno y el nitrógeno. ¿Qué presión habrá en cada una de las tres partes después que se establezca el equilibrio, si el sistema se mantiene a una temperatura constante de 300 K?

8.3-En un cilindro vertical con un área S de sección transversal y bajo un pistón de masa M se encuentra un gas ideal monoatómico. En cierto instante, bajo el pistón se enciende un calentador eléctrico que da al gas una cantidad de calor q por unidad de tiempo. Determine la velocidad v del movimiento del pistón bajo la condición de que la presión del gas bajo el pistón se mantiene constante y que el gas se expande contra la presión atmosférica, po. El gas bajo el pistón está térmicamente aislado.

8.4-Supongamos que a un planeta de masa M y radio ro lo rodea una atmósfera de densidad constante, compuesta por un gas de masa molar μ. Determine la temperatura T de la atmósfera sobre la superficie del planeta si el espesor de la atmósfera es h (h δq, lo que significa que (Q-δq) ≈ Q. O sea, la carga de la esfera grande prácticamente no varía cuando le cede algo de carga a cualquier esferita muy pequeña. Así, el potencial de la esfera en su superficie será todo el tiempo Vo = kQ/R, aunque intercambie carga con las esferitas.

Éste será también el potencial en la superficie de cada esferita, donde su carga no se distribuirá uniformemente por la superficie debido a la influencia de la carga de la esfera grande. Por otra parte, la carga adquirida por cada esferita es tan pequeña que no perturbará la distribución esférica de la carga en la esfera grande, y su potencial a cualquier distancia s de su centro seguirá dada por la ley V = kQ/s. Por otro lado, en cada esferita el potencial en su superficie es igual al potencial en todos los puntos de su volumen, y en particular, igual a su potencial en el centro (recordar que las cargas se distribuyen en la superficie del metal de forma tal que la intensidad del campo eléctrico sea nulo en su interior, por lo que todo el volumen debe estar al mismo potencial). Y el potencial en el centro de una esferita puede calcularse sumando los potenciales de todas las cargas sobre su superficie (a una distancia r del centro) con los potenciales de todas las cargas externas a la esferita: Vcentro = kΣqi superf /r + ΣViexternos donde r es el radio de la esferita (todas las cargas superficiales están a la misma distancia r del centro). Puede reescribirse: Vcentro = kq /r + ΣViexternos

(1)

donde q es la carga total en la esferita. Aplicaremos esta ecuación (1) a cada esferita, que siente el efecto de la esfera grande y el de las otras esferitas. Llamemos l a la distancia entre el centro de cada esferita y el centro de la esfera grande (lados inclinados de la pirámide en la figura). Sea r el radio de cada esferita y sea d la distancia de centro a centro de las esferitas (lados de la base triangular en la figura). Puesto que la carga Q de la grande prácticamente no varía en el proceso, su potencial tampoco cambia sustancialmente y las esferitas, puestas en contacto con la grande, tendrán en sus supericies el mismo potencial Vo producido por la esfera grande en su superficie.

97

Al conectar la primera esferita a la grande, el potencial Vo que adquiere será expresable de acuerdo con (Ec. 1): Vo = kq1/r + kQ/l (2) Al conectar la segunda esferita, su potencial será: Al conectar la tercera:

Vo = kq2/r + kQ/l + kq1/d Vo = kq3/r + kQ/l + kq1/d + kq2/d

De (2) y (3): q2/q1 = (1 – r/d)

(5)

De (2) y (4): q1(1 – r/d) = q2 r/d + q3

(6)

(3) (4)

Sustituyendo (1 – r/d) de (5) en (6) y reordenando: q3 = q2(1 – r/d) Empleando (5) nuevamente: q3= q22/q1

11.4-La diferencia de potencial que surge entre los extremos de cada varilla está ligada al campo eléctrico interior que surge de extremo a extremo en cada una, campo responsable de acelerar con cada varilla a los electrones de conducción de cada una. En el modelo de la conducción eléctrica en un metal, se asume la existencia de una nube de electrones libres dentro del metal (como un gas de electrones libres en un recipiente cuyas paredes son las propias fronteras de la pieza metálica). Cuando este “recipiente de electrones libres” se acelera, los electrones tienden a rezagarse, acumulándose un poco en la pared “trasera” de la varilla y produciéndose una carga positiva neta en la pared “delantera” por la ausencia de electrones; estas cargas netas en los extremos de la varilla producen el campo eléctrico necesario para que todos los electrones del interior se aceleren con la varilla y no se queden afuera. Aclaramos que los electrones no llegan a escapar del metal porque la salida de unos pocos de ellos generaría una carga positiva neta en la varilla, impidiendo el escape de otros electrones. La aceleración de cada varilla se calcula de forma inmediata aplicando la segunda ley de Newton a cada una. La tensión T del hilo acelera a la varilla horizontal hacia el borde de la mesa, cumpliéndose: T = ma (no hay fricción), y en la varilla vertical se cumplirá: mg – T = ma. En ambas ecuaciones se ha planteado la misma masa m para cada varilla y la misma aceleración (en magnitud). De estas ecuaciones resulta: a = g/2. Cada electrón en cada varilla se moverá con esta aceleración debido al campo eléctrico E1 y E2 surgido en el interior de cada varilla. En la varilla horizontal se tendrá, para cada electrón: eE1 = mea , de donde resulta: E1 = me a/e = meg/2e. En la varilla vertical se tendrá, para cada electrón: meg – eE2 = mea = meg/2. De aquí: Ee = meg/2e = E1.

98

Ambos campos son de igual intensidad: el horizontal acelera a los electrones y el vertical los frena sin dejarlos caer libremente por gravedad. Estos dos campos son uniformes dentro de cada varilla y a ellos aplica la ecuación E = −ΔV/Δr, de donde podemos calcular el ΔV entre los extremos de cada varilla. Despejando: ΔV= −EΔr = −megl/2e. Como el cociente me/e es del orden de 10-12 kg/C, esta diferencia de potencial entre los extremos de una varilla normal (digamos de 1 m de largo) es indetectable.

11.5-La capacitancia del capacitor podemos expresarla como C = εCo, donde Co es su capacitancia sin dieléctrico. Con dieléctrico no lineal, la capacitancia será: C = αVCo. Al ponerse en paralelo, ambos capacitores igualan los potenciales de sus placas en contacto, e igualan sus diferencias de potencial entre placa y placa, a un valor V. La carga en cada uno será: qvacío =VCo y qno lineal = αV2Co. Ahora: la carga total que hay para ambos capacitores es la que acumulaba inicialmente el capacitor vacío, qo = VoCo. Y por la ley de conservación de la carga: qno lineal + qvacío = qo. Sustituyendo cada carga por su expresión en función de los voltajes: αV2Co + VCo = VoCo. Cancelando Co se obtiene: αV2 + V − Vo = 0, y resolviendo esta ecuación cuadrática para V, se obtienen dos raíces, una positiva y otra negativa. La positiva es: V = 12 V.

11.6-No hay forma de conectar la batería y que se carguen todos los capacitores. Siempre hay uno que quedará entre dos potenciales iguales y no se carga. La manera más fácil de verlo es poniendo el circuito en forma tridimensional de tetraedro regular, o pirámide (Fig. 11.6S). Siempre habrá un capacitor en cada lado de la pirámide, y entre cualesquiera dos vértices que se conecte la batería, siempre entre los dos otros vértices quedará un capacitor cuyos dos extremos estarán a iguales potenciales, por lo que no se cargará. En la Fig.11.6S, el capacitor de la rama AB quedará sin cargar.

A

C D

B

99

Fig.11.6S

+ Batería

12A-Circuitos. Problemas. (El símbolo

representa un resistor)

12.1-El circuito de la Fig. 12.1 se compone de resistores con resistencias R1=1.0 Ω, R2= 4.0 Ω, R3=2.0 Ω y R4=3.0 Ω. ¿Qué corriente circula a través del amperímetro A1 si la corriente a través del amperímetro A2 es de 5.0 A? ¿Cuál es la lectura del voltímetro V? Considere ideales los instrumentos de medición. R1

R3

A1 A1

A2

A2

A3

R2

R4

A4

1

V

Fig. 12.1 11.2

Fig. 12.2

12.2-Cuatro amperímetros idénticos y un resistor están conectados tal como lo indica la Fig. 12.2. El amperímetro A1 marca I1 = 2.0 A, el A2, I2 = 3.0 A. ¿Qué corrientes pasan por los amperímetros A3 y A4 y por el resistor? Encuentre la relación r/R entre la resistencia interna r del amperímetro y la resistencia R del resistor.

12.3-Se monta un circuito de acuerdo con el esquema de la Fig. 12.3. El voltímetro se conecta en paralelo con el resistor de resistencia R1 =0.40 Ω y registra un voltaje V1 = 34.8 V. La tensión en los bornes de la fuente de corriente se mantiene constante e igual a 100 V. Encuentre la relación entre la intensidad de la corriente que pasa por el voltímetro y la de la que pasa por el resistor R2, de resistencia R2 = 0.60 Ω. C A E

100 V R1

E

R2

B

1

V1

E R

Fig. 12.4

Fig. 12.3

12.4-Encuentre cómo cambia el voltaje en el resistor R del circuito que se muestra en la Fig.12.4 en dependencia del valor de la resistencia R de dicho resistor. La fem de cada una de las fuentes es igual a E y sus resistencias internas son r. Considerar el diodo como ideal (resistencia cero en polarización directa e infinita en polarización inversa). 100

12.5-El elemento S posee la característica voltampérica representada en la Fig. 12.5 (es la característica idealizada de un estabilitrón). Se conectan en serie un resistor de resistencia R, el elemento S y una batería de fem E. ¿Cuál es la magnitud de la corriente en el circuito? I Elemento B

Elemento S

I Io V

-Io 0

Vo

V Fig. 12.6

Fig. 12.5

12.6-El elemento B posee la característica voltampérica mostrada en la Fig. 12.6. Se conectan en serie un elemento B, una batería de fem E y un resistor de resistencia R. ¿Qué tensión se produce en B?

12.7-Encuentre las corrientes a través de los elementos S y las tensiones en los elementos B de los circuítos mostrados en las Fig. 12.7(a) y (b), para el caso en que E > IoR + Vo. Las características voltampéricas de los elementos S y B se dan en los dos ejercicios anteriores. S

B

E S

E

R (a)

B

R (b)

Fig. 12.7

12.8-Calcule la resistencia entre los puntos A y B del circuito eléctrico infinito que se muestra en la Fig. 12.8, si todas las resistencias del circuito son idénticas.

B

R

R R

R

A Fig. 12.8

12.9-En el circuito de la Fig. 12.9 (próxima página) la fem (fuerza electromotriz) de la batería es de E = 100 V, su resistencia interna es r = 100 Ω, la capacitancia es de C = 200 μF y la resistencia es R = 10.0 Ω; esta resistencia representa un calentador. 101

R

C K 2

El interruptor K hace contacto entre los puntos 1 y 2 a razón de 10 veces por segundo. Cuando el interruptor se encuentra en la posición 1 el capacitor se carga, y al pasarlo a la posición 2, se descarga.

1

Calcule el rendimiento térmico de este circuito, η, y establezca cuántas veces mayor es este rendimiento (con el conmutador periódico) que si se conectara diE ,r rectamente la batería a la resistencia de calentamiento, sin capacitor. Fig. 12.9 [η = (potencia de calentamiento en la resistencia) / (potencia entregada por la batería)]

12.10-Determine la carga eléctrica que se acumula en el capacitor de la Fig.12.10: (a)Para el circuito tal y como se muestra. (b)Para el circuito, con el capacitor y el resistor de 200 Ω intercambiados. (c)¿Qué cantidad de electrones pasa por la batería en 10 min. en el caso (a)?

80 Ω

A 220 Ω B

180 Ω C

10 μF

15.0 V 120 Ω

200 Ω

Fig.12.10

D

K

12.11-En el circuito mostrado en la Fig. 12.11, R = 100 Ω, C = 10 μF, Vo =10 V, las resistencias internas de la batería y del amperímetro son despreciables. El interruptor K se cierra periódicamente por un tiempo t1 = 1×10-3 s y se abre por un tiempo t2 = 20×10-3 s. Bajo tal régimen, la aguja del amperímetro marca un valor prácticamente constante. ¿Qué valor marca el amperímetro? 12.12-En el circuito de la Fig.12.12, cierran el interruptor K. Encontrar la máxima corriente a través de la bobina de inductancia L en función de los datos de la figura. Encontrar la tensión máxima en el capacitor C1. Considere ideales los elementos del circuito (incluyendo batería recargable).

C R A

Vo C

R

Fig.12.11

L K C2

C1

Fig.12.12 102

Vo

12.13-En un circuito común rectificador de media onda (ver Fig.12.13): C = 1000 μF, R = 500 Ω. La frecuencia del circuito es f = 50 Hz. Considerando ideal el diodo encuentre: (a) El coeficiente de pulsaciones de voltaje, ΔV/Vmax, en el resistor. (b) Diseñe un circuito rectificador de onda completa con cuatro diodos ideales, y las mismas resistencia R y capacitancia C.

diodo R

C

Fig.12.13

12B-Circuitos. Soluciones.

12.1-En la Fig.12.1S se representan las corrientes I1, I2, I3, I4 e I5, y las direcciones que les suponemos. Aplicando la ley de los nodos al nodo P:

I3

I5 = I1 – I3.

I4

Como el amperímetro 1 está registrando una corriente finita (no inmensa, no en cortocircuito), y

R3

P

R1

I5 A1

T

I1 R2

R4 Q V

A2

5.0 A

I2

1

Fig. 12.1S

su resistencia interna es cero (amperímetro ideal), significa que la diferencia de potencial entre sus extremos P y Q es cero (una resistencia cero entre dos puntos a diferentes potenciales ocasionaría una corriente enorme, de cortocircuito, pues I = V/r). En este caso, las caídas de potencial en las resistencias R1 y R2 son iguales: I1R1 = I2R2, de donde resulta I1 = 4I2. Pero I1 + I2 = 5.0 A (que es lo registrado en el amperímetro A1). Entonces: I1 = 4.0 A e I2 = 1.0 A. Además, las caídas de potencial en R3 y R4 tienen que ser iguales también, pues representan las variaciones de potencial desde P y Q hasta un punto común T por el que se retiran las corrientes I3 e I4, que deben sumar 5.0 A nuevamente. Procediendo como se hizo con I1 e I2 se obtiene: I4 = 2.0 A e 103

I3 = 3.0 A. De los cálculos realizados se ve que por el amperímetro A1 debe pasar una corriente I5 = I1 – I3 = 1.0 A. Y el voltaje del voltímetro es: V = I1R1 + I3R3 = I2R2 + I4R4 = 10 V.

12.2-En la Fig.12.2S, se indican las direcciones que se le proponen a las corrientes, con I1=2.0 A e I2 = 3.0 A. La caída de potencial entre los nodos P y Q tiene que ser la misma, calculada por la suma de voltajes en los amperímetros A1 y A3, o calculada por la caída de voltaje del amperímeI1 tro A2: VPQ = I2r = I1 r + I3 r. P I2 De aquí: I3 = I2 – I1 = 1.0 A.

A1

A2

I3

S

A3

Aplicando la ley de nodos al nodo Q: R

I4 = I2 + I3 = 4.0 A.

Q I4

I

A4

T

Y en el nodo S: I = I1 – I3 = 1.0 A,

Fig. 12.2S

con lo cual quedan determinadas todas las corrientes. Para determinar la relación r/R, calculemos la diferencia de potencial entre S y T por la rama que contiene a R y por la que contiene a A3 y A4: VST = IR = I3 r + I4 r. De aquí: r/R = I/(I3 + I4) = 0.20

12.3-En R2 el voltaje que cae es V2 = E - V1, por lo que la corriente que pasa por esa resistencia es:

100 V R1

I2 = (E - V1)/R2

R2 1

V1

Fig. 12.3

(1)

A su vez, I2 = I1 + IV = V1/R1 + IV, donde IV es la corriente en el voltímetro. Despejando: IV = I2 – V1/R1.

Sustituyendo I2 por la expresión (1): IV = (E - V1)/R2 – V1/R1

(2)

De (1) y (2): IV/I2 = 1 – V1R2 / [(E - V1) R1] ≈ 0.20

12.4-Para encontrar la dependencia del voltaje V que cae en R, respecto al valor de la propia resistencia R, analizaremos el comportamiento del circuito para valores extremos de R (R grande y 104

C A E E

E

Fig. 12.4

E

r

E

E

I’

r

I

B IR

R

E

E r

IR

r

Fig. 12.4Sb

R

Fig. 12.4Sa

R

E

r

E

r

IR

I1

E

r

Fig. 12.4Sc I2

R

R pequeño) para tratar de detectar en qué rango de valores el diodo queda en polarización directa y en qué rango, en inversa. La Fig. 12.4Sa muestra el circuito con la posible corriente I’ por el diodo (cuando éste conduzca) y la corriente I por la rama de las dos baterías en serie. Las figuras 12.4Sb y 12.4Sc muestran cómo quedaría el circuito equivalente completo cuando el diodo no conduzca (caso b) y cuando conduzca (caso c). Consideremos primero altos valores de R (R>>r). En este caso la corriente total del circuito será baja, por lo que lo serán I e I’. Los potenciales entre los puntos A y B, señalados en la figura (a), se relacionarán: VB = VA + 2E – 2Ir ≈ VA + 2E. Y entre A y C: VC = VA + E – I’r – Vdiodo ≈ VA + E – Vdiodo. Pero: VC = VB, por lo que: VA + 2E = VA + E – Vdiodo. De aquí: Vdiodo = – E. Esto significa que el diodo está polarizado en inversa para altos valores de la resistencia R y no pasará corriente por la rama del diodo, I’ = 0, por lo que el circuito completo se reduce al equivalente de la Fig. 12.4Sb, en el que la corriente por la resistencia R es la misma que la de la rama con las dos baterías, I. Aplicando la ley de Ohm a este circuito: I(R + 2r) = 2E. Entonces el voltaje en la resistencia es: VR = 2E R/(R + 2r) ≈ 2E

(1) para R grande

Al analizar el otro extremo, R →0, aumenta la caída de potencial en las resistencias internas de las baterías en la rama AB, por lo que el potencial VB en B empieza a disminuir, y con él, VC, pues: VB = VA + 2E – 2Ir = VC Cuando 2Ir alcance el valor E se tendrá VB = VA + E y se llega al límite de la polarización inversa del diodo (Vdiodo= 0, con I’= 0). Aún en este límite, la corriente I’ es nula por lo que la corriente en la resistencia R es la misma que la de la rama AB, I, y sigue siendo válido el circuito

105

equivalente de la figura (b). Se cumplirá entonces: 2E – 2Ir – IR = 0, con 2Ir = E, por lo que: 4Ir – 2Ir = IR, de donde: R = 2r. O sea, con este valor de R se llega al límite de la polarización inversa del diodo. Para valores de R menores que 2r, el diodo comenzará a conducir y el circuito equivalente del sistema es el de la figura (c). Si R Vo, pasará corriente por el circuito, el voltaje en el elemento S será Vo e I = (E – Vo)/R. Pero si E < Vo, la resistencia del elemento se hace infinita y no pasa corriente: I = 0. Todo el voltaje cae entonces en el elemento S.

12.6- Si se conectan en serie el elemento B y la resistencia, y se cumple que IoR < E , circulará entonces la corriente Io que fija el elemento B, y: E = IoR +VB, de donde resulta: VB = E − IoR. Por otra parte, si IoR > E, el elemento B se hace conductor ideal, en el que no cae voltaje, VB = 0, y pasaría una corriente menor que Io, tal que I R = E.

12.7-En la Fig. 12.7(a) los elementos S y B están en serie con la batería. Se cumplirá:

E = IoR + Vo + VB. Así, queda fijada la corriente Io por el elemento B, y en S cae el voltaje Vo. El voltaje en B será: VB = E − IoR – Vo, si E > IoR + Vo. Cuando los elementos se conectan en paralelo, como indica la Fig. 12.7(b): VR = IR = E – Vo, ya que el elemento S fija el voltaje del paralelo. De aquí: I = (E − Vo) / R. 106

Pero I = Io + IS. Entonces: IS = (E − Vo) / R − Io, si E > IoR + Vo.

12.8-Como el circuito del problema es una cadena de infinitos eslabones idénticos, la resistencia RAB que exhibe entre los puntos A y B en la entrada del primer eslabón será la misma que la que exhibe entre los puntos análogos de la entrada del segundo eslabón, RAB, como

B

R R

RAB

A Fig. 12.8S

muestra la Fig. 12.8S. Pero la resistencia total de este segundo circuito es fácilmente calculable (es R en serie con el paralelo de R y RAB): RAB = R + RRAB/(R + RAB). Despejando: RAB = (1 +

R

C K 2

1

E ,r

Fig. 12.9

)/2 ≈ 1.62 R

12.9-Comencemos por señalar que el tiempo característico de carga del capacitor es ηc = (R + r) C = 0.022 s, y el tiempo de descarga será ηd = RC = 0.002 s. Por otro lado, cada ciclo dura 1/10 = 0.1 s, que equivale a unos 5 tiempos ηc de carga y unos 50 tiempos ηc de descarga, por lo que pueden considerarse cargas y descargas completas en cada ciclo con error menor del 1%.

Bajo el proceso de carga, con el interruptor K en 1, el capacitor alcanza un voltaje V =E y una carga q = CE. El trabajo realizado por la batería en mover toda esa carga será W = qE = CE 2. La energía acumulada en el capacitor será U = ½ CE 2, que es la mitad de lo trabajado por la batería. Durante esta carga, el calor liberado en la resistencia de calentamiento es: Q1 = Σ Ii2RΔti = R Σ Ii2Δti. Y el calor liberado en la resistencia interna de la batería es: Q2 = Σ Ii2rΔti = r Σ Ii2Δti. Entonces: Q1/Q2 = R/r

(1)

Además, este calor total liberado representa la otra mitad del trabajo hecho por la batería: Q1 + Q2 = ½ CE 2

(2) 107

De (1) y (2): Q1 = ½ CE 2 R/(R + r)

(3)

Este es el calor liberado en la conexión de K con 1. Al pasar el interruptor K a 2, se descarga el capacitor y libera en la resistencia R toda la energía que había acumulado: U = ½ CE 2, por lo que en el ciclo completo de carga y descarga, el calor liberado en la resistencia de calentamiento R es: Q = ½ C E 2 [1 + R/(R + r)] El rendimiento será: η = Pcalent /Pbatería = Q(1 ciclo)/W(1 ciclo) = ½[ 1 + R/(R + r)] = 0.54

(A)

Si se conecta la resistencia directamente a la batería, sin capacitor ni conmutador, la corriente sería: I = E / (R + r) y la potencia de la batería sería: Pbat = IE = E 2/(R + r). La potencia en la resistencia sería: Pcalent = I 2R = E 2R / (R + r)2. Entonces: η’ = Pcalent /Pbatería = [E 2R / (R + r)2] / [E 2/(R + r)] = R/(R + r) = 0.09

(B)

O sea, el circuito con carga intermitente es 6 veces más eficiente: 0.54/0.09 = 6.

12-10-(a)Cuando el capacitor se cargue no pasará corriente de B a C, por lo que el puente será equivalente a dos ramas en paralelo, de resistencias: Rabd = 340 Ω y Racd = 380 Ω La diferencia de potencial entre B y C será el voltaje de carga del capacitor. Para determinar este voltaje necesitamos las corrientes en cada rama del paralelo. La resistencia total del paralelo, R, satisface:

A 80 Ω

220 Ω B

15.0 V

1/Rp = 1/340 + 1/380 = 0.005573, por lo que Rp = 179.4 Ω.

180 Ω C

10 μF

120 Ω

200 Ω D

La resistencia total del circuito será: Fig.12.10

RT = 80 + 179.4 = 259.4 Ω La corriente que sale de la batería es: Ib = E /RT = 15.0/259.4 = 57.82×10-3 A = 57.82 mA El voltaje en el paralelo es: Vad = IbRp = 0.05782×179.4 = 10.37 V La corriente en cada rama es: Iabd = Vad/Rabd = 10.37/ 340 = 30.51×10-3 A = 30.51 mA y

Iacd = Vad/Racd = 10.37/ 380 = 27.30×10-3 A = 27.30 mA 108

La caída de voltaje desde A hasta B es: Vab = 220 Iabd = 6.71 V Y la caída de voltaje desde A hasta D es: Vac = 180 Iacd = 4.91 V Entonces el voltaje entre B y C es: Vbc = 6.71 – 4.91 = 1.8 V Y la carga en el capacitor será: q = CVbc = 10×1.8 = 18 μC b)Intercambiando el capacitor con la resistencia de 200 Ω queda un circuito equivalente al que muestra la Fig. 12-10S. En él, las resistencias de 180 Ω y 200 Ω quedan en serie, para una resistencia equivalente en serie igual a 380 Ω. Estas dos quedan en paralelo con la de 220 Ω, para una resistencia equivalente en paralelo de 139.3 Ω. Todo este paralelo estaría en serie con las resistencias de 80 Ω y 120 Ω, para una resistencia total en el circuito de 339.3 Ω. El voltaje de carga del capacitor sería ahora la diferencia de potenciales entre los puntos C y D.

80 Ω

A 180 Ω

220 Ω B 15.0 V

200 Ω

120 Ω

C 10 μF

D

Fig.12.10S

La corriente de la batería es ahora: Ib = 15.0/339.3 = 44.2 mA. El voltaje en el paralelo será: Vp = 0.04421×139.3 = 6.158 V. La corriente en la rama ACB será: Iacb = 6.158/380 = 1.62 mA; y la corriente en la rama BD es la misma que en la batería, Ibd = 44.2 mA. Entonces: Vcd = Vcb + Vbd =0.00162×200 + 0.0442×120 = 5.6 V La carga en el capacitor será: q = CVcd = 56 μC c)La corriente de la batería en el caso (a) es Ib = 57.8 mA. La corriente se define como I = Δq/Δt, pero se cumple que Δq = Ne (la carga es siempre un múltiplo entero de la carga del electrón), por lo que: I = Ne/Δt Y: N = IΔt/e = 0.0578×600/(1.60×10-19 = 2.171020 electrones

12.11-La solución de este problema es rápida si notamos que el ciclo de carga se logra sin pasar prácticamente por los resistores (ciclo de carga casi instantáneo, pues RC→0), como muestra la Fig.12.11Sa (próxima página). En tanto el ciclo de descarga se produce a través de los resistores, como muestra la Fig.12.11Sb. En el ciclo de carga, el voltaje que adquiere cada capacitor es Vo/2, de manera instantánea, 109

K

C

+ −

A

Vo C

+ −

Fig.12.11Sa

y la carga que acumula cada capacitor es qc= CVo/2, que es la carga que circula por el amperímetro por causa de los capacitores. Ahora, las resistencias en paralelo con cada capacitor adquieren también instantáneamente el voltaje Vo/2, y con ello se produce una corriente directa sobre las resistencias de valor IR = Vo/ (2R), que en el amperímetro es opuesta a la corriente de carga de los capaci-

+

C



R A C

+ −

R

Fig.12.11Sb

tores. Como el interruptor está cerrado un tiempo t1, la carga que pasa por el amperímetro, por causa de los resistores es: qr = IRt1 = Vot1/(2R). La carga total que pasa por el amperímetro durante el tiempo en que el interruptor K está cerrado es: q1 = qr – qc = Vot1/(2R) − CVo/2 donde consideramos positiva la corriente hacia la derecha en el amperímetro. (El análisis realizado se basa en el principio de superposición: calcular qué carga pasa por el amperímetro debido a una u otra causa, y luego se superponen las cargas por una suma algebraica). En el ciclo en que se abre el interruptor K, los capacitores se descargan, cada uno a través de una resistencia, formando dos lazos de descarga que envían corriente en la misma dirección sobre el amperímetro, dirección que coincide con la que consideramos positiva anteriormente. El tiempo en que cada capacitor se descarga en un 63% es η = RC = 100×10×10-6 = 1×10-3 s, que es 20 veces menor que el tiempo t2 con el interruptor abierto, lo cual quiere decir que los capacitores se descargan totalmente en cada ciclo a los efectos prácticos. Como los dos capacitores envían sus cargas en la misma dirección por el amperímetro, la carga total que pasa por éste durante el tiempo t2 es: q2 = 2qc = CVo El valor constante que marcará el amperímetro será el de la corriente media en el ciclo completo de duración t1 + t2. O sea: Im = (q1 + q2) / (t1 + t2) = [Vot1/(2R) − CVo/2 + CVo] / (t1 + t2) = [Vot1/(2R) + CVo/2] / (t1 + t2) Sustituyendo valores: Im = 4.8×10-3 A = 4.8 mA El ciclo entero dura t1 + t2 = 21×10-3 s = 21 ms, y en ese tiempo la inercia mecánica de la aguja del amperímetro no le permite ir y volver a cero; apenas vibrará imperceptiblemente cerca del valor de corriente media.

110

12.12- Antes de cerrar K, los dos capacitores están en serie, cargados, y la carga en los dos capacitores es la misma, pues se cargan estando en serie. La capacitancia total de ambos en serie es:

L K C2

Ceq= C1C2/ (C1 + C2) C1

Y la carga en cada uno es: qo = Vo C1C2/ (C1 + C2) Fig.12.12 2 La energía acumulada entre los dos capacitores es: Uo = ½ Vo C1C2/ (C1 + C2)

Vo

Al cerrar el interruptor, los dos capacitores y la batería mandarán carga a través de la bobina. La máxima corriente que pase por la bobina provocará que en ese momento no haya fem en la misma, pues E = - L dI/dt (en el máximo de la función I la derivada es cero). En ese momento el capacitor C2 debe haberse descargado totalmente (ya no tiene mas carga que enviar y la corriente llegó a su máximo). En ese instante, el voltaje en C1 es Vo (ya que C2 está descargado y en la bobina la fem es cero), por lo que la energía total en el circuito en ese instante es: U = ½LI2max + ½C1Vo2 La diferencia entre esta energía y la inicial es: ΔU = ½LI2max + ½C1Vo2 − ½ Vo2 C1C2/ (C1 + C2) Este aumento de energía en el circuito se debe al trabajo hecho por la batería mandando carga al circuito, ΔU = VoΔq, donde el Δq se mide por la diferencia entre la carga inicial en los capacitores y la carga existente en C1 en ese instante: Δq = C1Vo - Vo C1C2/ (C1 + C2) Entonces: ½LI2max + ½C1Vo2 − ½ Vo2 C1C2/ (C1 + C2) = Vo [C1Vo - Vo C1C2/ (C1 + C2)] Despejando: Imax = C1Vo/[L(C1 + C2)]1/2 lo cual contesta la primera pregunta. Para contestar la segunda, notemos que C1 y C2 están en una misma malla, por lo que en todo momento: Vo = V1 + V2. Esto implica que cuando V1 sea máxima, V2 será mínima, pues Vo es constante. Además, en el contorno L-C2 se tendrá que cuando V2 esté en un valor extremo (máximo o mínimo), en L estará ocurriendo una inversión de corriente, lo que significa que I = 0 en esos instantes; y por conservación de la carga en el nodo, implica que tampoco en C1 hay corriente en ese momento. Entonces, por conservación de la energía para el instante en que V1 es máximo: ½C1V12 + ½C2(Vo – V1)2 = ½ Vo2 C1C2/ (C1 + C2) + Wbat 111

donde ahora: Wbat = VoΔq’ = Vo [C1V1 - Vo C1C2/ (C1 + C2)] Sustituyendo en la ecuación anterior y arreglando: V12 (C1 + C2)/2 – V1 (C1+ C2) Vo + [½Vo2 (C22 + 2C1 C2 ) / (C1 + C2 ) ] = 0 Resolviendo la cuadrática: V1 = [(C1 + C2) Vo ± C1Vo] / (C1 + C2) Con soluciones: V1,max = Vo[1 + C1/(C1 + C2)] y V1,min = Vo[1 − C1/(C1 + C2)]

12.13-Para los valores dados en el circuito, el tiempo característico de descarga del capacitor será: η = RC = 0.5 s

diodo

Y el período de la señal alterna será: T = 1/f = 0.02 s C

O sea, se cumple: η >> T (25 veces mayor).

R

El tipo de voltaje que caerá en el resistor tendrá la forma Fig.12.13 aproximada mostrada en la Fig. 12.13Sa, donde se ve que el tiempo que está descargándose el capacitor en cada ciclo es casi igual al período del ciclo (el tiempo de carga es muy breve en cada ciclo). Si ΔVV2εoS / lo3.

16.5-La diferencia entre las dos situaciones está en que en el primer caso las placas se cargan bajo una diferencia de potencial E, con la capacitancia que haya cuando la placa móvil se estabilice (q1 = C1E ), y en el segundo caso, la carga se adquiere con una diferencia de potencial E, con la capacitancia inicial de las placas a la distancia d (q2 = C2 E ).

154

A B

d

resorte

pared

pared K

Fig.16.5

Para el primer caso: q1 = C1E = [εoS/(d – x1)]E donde S es el área de cada placa y x1 = 0.10d es el corrimiento de la placa hasta su nueva posición de equilibrio. El campo total entre las placas será: E1 = E / (d – x1). La mitad de este campo es la producida por cada placa separadamente, y para calcular la fuerza que experimenta una placa en su atracción por la otra necesitamos la carga de una placa, q1, y el campo que experimenta por parte de la otra. Así, el campo que experimenta la placa móvil E = E1/2 = E / [2(d – x1)]. Entonces, la fuerza de atracción de una placa sobre la otra es: F1 = q1E = {[εoS/ (d – x1)] E } {E /[2(d – x1)]} = εoS E 2/ [2(d – x1)2]. Esta fuerza queda compensada por la del resorte: F = kx1: kx1 = εoSE 2/ [2(d – x1)2]

(1)

En esta expresión tenemos una relación entre la deformación del resorte, x1, y los parámetros del sistema para el primer caso. Trataremos ahora de hallar una relación similar en el segundo caso, con una compresión diferente, x2, y poder establecer la relación entre ambas deformaciones. En el segundo caso: q2 = C2E donde C2 = εoS/d. Cuando la placa móvil alcance el equilibrio de fuerzas ya no cambiará esta carga q2 porque la batería ya se desconectó. El campo total en el equilibrio es ahora E2 = ζ/εo = q2/ (Sεo), y el campo de la placa fija sobre la móvil será E2/2. La fuerza sobre la placa móvil es ahora: F2= q2E2/2 = q22/(2εoS) = C22E 2/(2εoS) . Sustituyendo C2: F2= εoSE 2/(2d2). Esta fuerza de atracción eléctrica estará en equilibrio con la fuerza elástica: kx2 = εoSE 2/ (2d2)

(2)

Dividiendo miembro a miembro la ecuación (1) por la (2):

x1/x2 = d2/ (d – x1)2

Despejando: x2 = x1(d – x1)2/d2 con x1 = 0.10d. Así: x2 = 0.08d.

16.6-Si el resorte colgara estáticamente, en equilibrio, se estiraría una longitud Δl = mg/k. En la Fig.16.6S (próxima página) se representa un momento cualquiera en el que el punto de suspensión se ha desplazado Asenωt y la esfera se ha corrido una distancia x respecto a su posición de equilibrio estático. En el equilibrio estático, el resorte tendría sus extremos entre los puntos O y P. El punto O indica el centro de oscilación del punto de suspensión móvil, y lo sería la longitud del resorte sin deformar (sin peso colgado). Se cumplirá para la esfera: mg – kΔx = m (d2x/dt2), donde Δx = mg/k + x + A senωt (representa la deformación total del resorte). Sustituyendo Δx en la ecuación anterior se obtiene: 155

Suspensión móvil

m (d2x/dt2) = −kA senωt – kx Asenωt O lo+mg/k

P x m

(+)

Fig. 16.6S

Arreglando: (d2x/dt2) + (k/m)x = −(kA/m) senωt

(1)

Ésta es la ecuación del oscilador armónico simple “forzado” (está sometido a una fuerza periódica externa aplicada en su punto de suspensión). Esta ecuación tiene una solución compuesta de dos partes: la de la oscilación propia (natural) del oscilador, con frecuencia ωo = (k/m)1/2, y la de la oscilación estacionaria que impone la fuerza externa con su frecuencia ω. Las dos oscilaciones se superponen, pero solamente nos interesa la estacionaria (así nos lo piden en el problema). Se sabe que la oscilación estacionaria se realiza con la frecuencia impuesta por la fuerza externa, y con algún posible desfasaje respecto a la fuerza: xs = Psen (ωt +θ).

Nos corresponde evaluar la amplitud P y el desfasaje θ. Para ello, derivamos la solución estacionaria, xs, dos veces y la sustituimos en la ecuación (1): dxs/dt = Pω cos(ωt + θ) y d2x/dt2 = −Pω2 sen(ωt + θ) Sustituyendo en (1): −Pω2 sen(ωt + θ) + (k/m) P sen(ωt +θ) = −(kA/m) senωt Arreglando: P[(k/m) – ω2]sen(ωt +θ) = −(kA/m) senωt. Como esta ecuación debe cumplirse para todo valor de t esto obliga a que θ = 0 (solamente así ambos miembros serían cero a la vez, y máximos a la vez). Por otra parte, deberá cumplirse también que la amplitud que acompaña al seno sea la misma en ambos miembros: P[(k/m) – ω2] = −(kA/m), de donde: P = (Ak/m)/( ω2 – k/m) Pero k/m = ωo2, como ya dijimos antes. Entonces, la amplitud de la oscilación estacionaria será: P = Aωo2/(ω2 – ωo2) = A/(ω2/ ωo2 – 1) y la oscilación estacionaria quedará descrita por la ecuación: xs = A sen ωt / (ω2/ ωo2 – 1) Puede notarse que si la frecuencia de la fuerza externa coincide con la propia del sistema, la amplitud de oscilación tiende a infinito, que es el fenómeno de la resonancia, sin resistencia mecánica en el sistema.

16.7-En la Fig. 16.7S (próxima página) se ha dibujado un eje x positivo hacia abajo, con el origen O en el nivel de la posición de equilibrio de la varilla con los resortes (línea quebrada horizontal). En esa posición ya los resortes están algo estirados respecto a su longitud natural. 156

Soporte

×

×

×

×

×

×

×

×

×

×

×

×

×

×

×

×

×

×

×

×

×

B ×

×

×

×

×

×

× l×

×

×

×

×

×

×

×

×

k

× × × × × Halado hacia abajo × × × × ×

El peso mg de la varilla se compensa con la fuerza elástica total de los dos resortes, 2k Δlo, donde Δlo es el estiramiento en esas condiciones:

Soporte

C

k

×

mg = 2k Δlo

(1)

Por lo que Δlo = mg / (2k) Δlo

Fig. 16.7S

O

x

Sin campo aplicado B el resorte oscilaría de tal modo que en cada posición de la varilla se cumpliría mg – 2k Δl = ma, con Δl = Δlo + x, y x estaría medida desde la posición de equilibrio O. Así: mg – 2k (Δlo + x) = ma

(2)

Y teniendo en cuenta (1): −2kx = ma, de donde resulta d2x/dt2 + (2k/m) x = 0, que es la ecuación del oscilador armónico simple con frecuencia ω =

(3)

Al poner un campo magnético B en la zona, cuando la varilla suba y baje, hará que el flujo magnético varíe dentro del circuito cerrado (se crea un lazo de área variable en el campo magnético). La fuerza electromotriz (fem) así generada es E = −Blv, de acuerdo con la ley de Faraday. Esta fem es el voltaje aplicado al capacitor, por lo que la carga q en el mismo, en cada instante, será: q = CV = CE = −CvBl Como la velocidad es variable durante la oscilación, la carga en el capacitor lo será también. La corriente en el circuito será justamente la variación de carga en el capacitor por unidad de tiempo: I = dq/dt = −CBl dv/dt = −CBl a donde a es la aceleración de la varilla. La fuerza magnética sobre la varilla en cada momento será: Fmagn = BIl = −CB2l2a La fuerza total sobre la varilla bajo el campo magnético cambia ahora de la expresión (2) a: ma = mg – 2k(Δlo + x) − CB2l2a Teniendo en cuenta de nuevo la ecuación (1) y reagrupando términos, obtenemos: d2x/dt2 + [ 2k / (m + CB2l2 )] x = 0 lo cual corresponde a una oscilación armónica de frecuencia ω = 157

(4)

La varilla realizará una oscilación de frecuencia menor que la oscilación libre, según se aprecia de comparar (3) y (4). La fuerza magnética que aparece aumenta la inercia efectiva del sistema, de m a (m + CB2l2).

16.8-Sea l1 la longitud del resorte que une con el eje a la esfera más cercana, y l2, la longitud del resorte que une a las dos esferas. Si las esferas realizan circunferencias, se cumple para cada una la segunda ley de Newton (para fuerzas centrípetas): mω2l1 = k(l1 – lo) − k(l2 – lo)

mω2(l2 + l1) = k(l2 – lo)

y

De aquí: l1 = lo / [1 – 3mω2/k + (mω2/k)2] y l2 = (1 − mω2/k) lo / [1 – 3mω2/k + (mω2/k)2] Estas soluciones poseen sentido físico solamente si: 1 – 3mω2/k + (mω2/k)2 > 0 (1)

y (1 − mω2/k) ≥ 0

(2)

Llamemos: x = mω2/k (x > 0). De (2): 1 – x ≥ 0, o sea: 1 ≥ x > 0 De (1): x2 – 3x + 1 > 0, de donde resultan: x > (3 +

)/2 y x < (3 −

Y teniendo en cuenta (3) solamente es posible: 0 < x < (3 − O sea: 0 < mω2/k < (3 − ferencias es ω < [(k/m) (3 −

(3) )/2

)/2.

)/2. Significa que la condición para que las esferitas realicen circun)/2]1/2

158

17A-Problemas con Análisis y Cálculos Gráficos.

17.1-La Fig. 17.1 muestra dos gráficos, el primero de posición contra tiempo y el segundo de velocidad contra tiempo. En cada gráfico aparecen las curvas correspondientes a dos bicicletas, A y B en el gráfico 1 y C y D en el gráficoa 2. a)¿Se puede decir en qué instante se encuentran las bicicletas A y B en el primer gráfico? ¿Se puede decir en qué instante se encuentran las bicicletas C y D en el segundo gráfico? Si es posible, diga en cada caso en qué instante es el encuentro. b)¿Cuál tiene mayor rapidez en el instante t = 10 s en el primer gráfico? ¿Y en el segundo? c)¿Cuál tiene mayor aceleración, en valor absoluto, en t = 10 s en el primer gráfico? ¿Y en el segundo? d)¿Cuál recorrió la mayor distancia entre t = 0 y t = 10 s en el primer gráfico? ¿Y en el segundo? e)En t = 10 s, ¿A y B viajan en direcciones opuestas o en la misma dirección? ¿Y C y D? x(m)

v(m/s)

A

C

D

B

0

5

10

t(s)

0

[1]

5

10

t(s)

[2] Fig. 17.1

M

m

m

1

2

3

17.2-Una bola de masa M incide desde la izquierda sobre un sistema que consiste de dos bolas de masa m cada una, en reposo sobre una superficie horizontal lisa y unidas por un resorte ligero (Fig. 17.2). El choque es frontal y absolutamente elástico. Encuentre aproximadamente la relación de masas m/M = γ bajo la cual el choque ocurre al menos una vez más.

Fig. 17.2 159

17.3-Un pistón móvil divide un cilindro en dos partes de iguales volúmenes, Vo =1.0 ×10-3m3. En una parte hay aire seco y en la otra, vapor de agua con una masa de 4.0 g de agua líquida. Se calienta lentamente el cilindro y el pistón comienza a moverse. Después de un corrimiento equivalente a la cuarta parte de la longitud del cilindro, el movimiento del pistón cesa. ¿Cuánta masa de aire, Mo, y de vapor de agua, M, había en cada parte del recipiente antes del calentamiento? ¿Cuál era la temperatura inicial To y a qué temperatura T1 dejó de moverse el pistón? (Representaremos por t a la temperatura en grados Celsius; represente por T a la temperatura en kelvin). La masa molar del aire seco es de unos 29 g/mol y la del agua es 18 g. La dependencia de la presión del vapor saturado Ps(t) del agua con la temperatura está dada en la siguiente tabla: t (oC) Ps(kPa)

100 100

120 200

133 300

152 500

180 1000

17.4-En un vaso con agua introdujeron un calentador eléctrico y midieron la dependencia de la temperatura T del agua con el tiempo t, registrándose la siguiente la tabla: t(min) 0 T(oC) 20.0

1 26.2

2 31.8

3 36.8

4 41.4

5 45.6

6 49.3

7 52.7

8 55.8

9 58.5

10 61.1

(a)¿En cuántos grados centígrados se enfría el agua en 1 min si el calentador se desconecta a una temperatura del agua de 50oC? (b)¿Comenzará a hervir el agua si el calentador no se desconecta durante un tiempo suficientemente largo? Considere que la potencia de calentamiento no varía.

17.5-El extremo inferior de un tubo capilar de radio r = 0.20 mm y longitud l = 8.0 cm se sumerge en agua a una temperatura constante de Ti = 0oC. La temperatura de su extremo superior es de Ts = 100oC. ¿A qué altura h se eleva el agua en el capilar? Considere que la conductividad térmica del capilar supera en mucho a la conductividad térmica del agua. El intercambio de calor con el aire circundante se desprecia. El agua moja bien el capilar. Indicación: utilice la siguiente dependencia de la temperatura y la tensión superficial del agua: T (oC)

0

20

50

90

ζ (×10-3 N/m)

76

73

67

60

160

T(oC) 3

17.6-En un balde se encuentra una mezcla de agua y hielo de masa m = 10 kg. El balde lo llevaron a una habitación y comenzó a cambiar la temperatura de la mezcla. La dependencia de la temperatura con el tiempo está representada en la Fig. 10.6. El calor específico del agua es igual a ca = 4.2 kJ/(kg.K) y el calor de fusión del hielo es Lf = 340 kJ/kg. Determine la masa de hielo en el balde cuando se llevó para la habitación. La conductividad del balde se desprecia.

2 1

20

40

60

t(min)

Fig.17.6

17.7-En una cacerola no muy grande, fina y metálica, echaron 0.50 litros de agua y la colocaron sobre una hornilla. Midiendo la temperatura del agua en diferentes instantes construyeron el gráfico de dependencia de la temperatura con el tiempo. Después botaron el agua y en el mismo recipiente echaron 0.70 litros de alcohol. De nuevo colocaron la cacerola en la misma hornilla y repitieron las mediciones de temperatura y tiempo e hicieron el gráfico de temperatura contra tiempo para el alcohol. Ambos gráficos se representan en la Fig. 17.7. Determine mediante estos gráficos el calor específico del alcohol y su calor de ebullición si en 23 minutos de ebullición la cantidad de alcohol en la cacerola disminuyó a la mitad. La capacidad calorífica de la cacerola es de 200 J/K. La evaporación desde la superficie de los líquidos es despreciable. (Densidad del alcohol: 0.80 g/cm3)

T(oC) 100 80 00 60 40 20 t(min) 10

20

30

40

50

Fig. 17.7

17.8-Los géiseres pueden considerarse como grandes reservóreos subterráneos llenos de agua, calentada por el calor de la Tierra (Fig.17.8a, próxima página) La salida desde ellos a la superficie terrestre se realiza por un estrecho canal, el cual, en períodos de calma, está completamente lleno de agua. Considerando que el período activo se presenta cuando en el reservóreo comienza a hervir el agua, y que en el momento de erupción el canal está lleno solamente de agua que es expulsada al exterior, valore qué parte del agua pierde el reservóreo del géiser durante el tiempo de una erupción. La profundidad del canal es h = 90 m; el calor de vaporización del agua es Lv = 2.26×106 J/kg. La dependencia de la presión del vapor saturado con la temperatura está dada por el gráfico de la Fig.17.8b (próxima página).

161

P(MPa) 1.6 90 m 1.2 0.8 0.4 0

100

180

140

Fig. 17.8a

220 T(oC)

Fig.17.8b

I(A)

17.9-¿Podrían dos lámparas incandescentes de 60 W y 100 W, calculadas para trabajar a 110 V, conectarse en serie a 220 V de tensión, sabiendo que la sobrecarga admisible en cada lámpara no puede ser mayor que el 10% del valor nominal? La característica voltampérica de la lámpara de 100 W se muestra en la Fig. 17.9.

0.8 0.6 Fig.17.9

0.4 0.2

20

40

60

80 V (V)

17.10-El voltaje de trabajo de una lámpara, cuya característica voltampérica está dada en la Fig. 17.10, es igual a 3.5 V. La corriente se corta a un voltaje de 4.0 V, pues la lámpara se quema. a) Dos lámparas como ésta se unen en serie y se conectan a una fuente de voltaje que provee 5.0 V. ¿Qué corriente circula por las lámparas?

I(A)

Relación I-V para la lámpara

0.30 (Resistencia no lineal) 0.20

0.10 V(V)

b) ¿Qué resistencia es necesario conectar en paralelo a una de las lámparas para que el voltaje en la otra sea de 3.5 V?

1.0

2.0

3.0

4.0

Fig. 17.10

c) ¿Se quemará alguna de las lámparas si se cambia el resistor anterior por una tercera lámpara igual a las dos primeras? 162

I(A)

17.11-En el gráfico de la Fig. 17.11 se presenta la dependencia de la intensidad de corriente que pasa por una lámpara con la tensión Vl aplicada a ella. La lámpara se conecta a una fuente de voltaje constante de 10 V, en serie con un resistor de 4.0 Ω. Determine la potencia de la lámpara.

2.0

1.0

Fig. 17.11 10

5

Vl (V)

17.12-En la Fig. 17.12a se muestra la característica voltampérica del bombillo L de una linterna de bolsillo. El bombillo se conecta al circuito mostrado en la Fig. 17.12b. (a)Encuentre gráficamente la corriente en la lámpara L. (b)¿Pará qué posición del punto móvil del potenciómetro P, el voltaje entre los puntos A y B será nulo? (O sea, ¿cómo debe quedar dividida la resistencia de 40 Ω por el contacto móvil para que el voltaje entre los puntos A y B sea nulo?). (c)¿Para qué posición del punto móvil del potenciómetro el voltaje entre los puntos A y B prácticamente no cambia bajo pequeñas fluctuaciones de la fem de la batería? I(A) 0.4

r= 10Ω

0.3

A

0.2 0.1 0

L 0

1

2

3

4

5

B

R= 40Ω

E= 4 V

P

V(V)

Fig.17.12a

Fig.17.12b

17.13-Para obtener un voltaje que dependiera poco de la temperatura se montó el circuito mostrado en la Fig.17.13a (próxima página). La característica voltampérica del diodo D y su dependencia con la temperatura del medio circundante se muestra en la Fig.17.13b para T1= 125oC, T2= 25oC y T3= −60oC. El voltaje de la fuente es V =6.0 V para una temperatura de 25oC, y con el aumento de temperatura crece linealmente. El coeficiente de cambio del voltaje con la temperatura es igual a 25×10-3 V/ oC. Encuentre el voltaje entre los terminales A y B para T =25oC y la dependencia de este voltaje con la temperatura.

163

V

R1=5.0 kΩ A

I(mA) 1.5

T1 T2 T3

D 1.0 R2=0.2 kΩ 0.5

B

Fig. 17.13a

0.4

0.8

1.2 Vd(V)

Fig. 17.13b

17A-Problemas con Análisis y Cálculos Gráficos. Soluciones.

17.1-a)En el gráfico 1, en t = 10 s están en la misma posición. En el gráfico 2 se puede decir solamente que tienen la misma velocidad en t = 10 s, pero se desconoce dónde están las bicicletas en cualquier instante. x(m)

v(m/s)

A

C

D

B

0

5

10

t(s)

0

[1]

5

10

t(s)

[2] Fig. 17.1

b)En 1, la inclinación de la recta A es mayor, por lo que esa bicicleta tiene mayor rapidez en t = 10 s. En el gráfico 2, ambas bicicletas tienen la misma velocidad en t = 10 s, por lo que tienen la misma rapidez. c)En el primer gráfico las bicicletas tiene ambas velocidades constantes, o sea, no tienen aceleración, y en el segundo, C tiene mayor aceleración, pues su pendiente es mayor.

164

d)En el primer grafico, en el intervalo de tiempo considerado, el desplazamiento de B es mayor que el de A (se “lee” el Δx sobre el eje x para cada bicicleta), y como son movimientos sin retrocesos para cada uno, la distancia recorrida por B es mayor que la de A. En el segundo gráfico, la rapidez de la bicicleta D desde t = 0 hasta t = 10 s es mayor que la rapidez de C, por lo que la distancia recorrida por D es siempre mayor que la recorrida por C en cualquier subintervalo de tiempo entre t = 0 y t = 10 s, y así será para el intervalo completo. (Note que aquí se pide comparar distancias recorridas, no desplazamientos, que estarían dados por las áreas). e)En el gráfico 1, todo el tiempo van en direcciones opuestas, pues las pendientes (velocidades) tienen signos opuestos. En el 2, van en la misma dirección en t = 10 s, pues ambas tienen velocidades positivas (entre t = 0 y t = 5 s iban en direcciones opuestas).

17.2-La solución será solamente aproximada, como se pide en el enunciado. Para que haya un segundo choque deberá cumplirse que M > m, pues de lo contrario la bola 1 rebotaría en el choque y jamás alcanzaría a la segunda en otro choque. O sea, m/M < 1, pero debemos precisar mejor hasta cuánto debe disminuir el cociente m/M para asegurar otro choque. Después de chocar 1 con 2, la 1 sigue con una velocidad constante y el centro de masas (CM) de 2 y 3 se mueve con otra velocidad constante, mientras 2 y 3 realizan oscilaciones armónicas respecto al CM. Es necesario encontrar la ley de movimiento de las esferas 1 y 2 después del choque y ver bajo qué condiciones vuelven a coincidir (un nuevo choque). Supondremos instantáneo el choque (equivale a asumir que el tiempo de choque es mucho menor que el período de oscilación de las esferas). Bajo esta suposición se puede analizar el choque entre 1 y 2 como un choque entre dos esferas libres, para las que aplican las leyes de conservación del momentum y de la energía mecánica: Mvo = Mv1 + mv2

(1)

y

De (1) y (2): v1 = v0 (M – m) / (M + m)

½ Mvo2 = ½ Mv12 + ½ mv22

(2)

y v2 = 2vo / (M + m)

O llamando γ = m/M: v1 = v0 (1 – γ) / (1 + γ)

y v2 = 2vo / (1 + γ)

(3)

Asumiendo que el choque ocurrió en x1 = x2 = 0, la coordenada x1 de la bola 1 (que continúa libremente) será después del choque: x1 = v1t = v0 t (1 – γ) / (1 + γ)

(4)

Por su parte, el CM del sistema de las bolas 2 y 3 se moverá con velocidad: vcm = v2/2 = vo/(1 + γ) La coordenada del CM después del choque será: 165

xcm = xcm,0 + v2t = xcm,0 + vot /(1 + γ)

(5)

Respecto al CM de 2 y 3, tomado como sistema de referencia inercial, la bola 2 se movería según una ley: x2’= A sen ωt - xcm,0 donde ω = (k/m)1/2, con k como la constante de medio resorte (desde la bola 2 hasta el centro del resorte completo). La amplitud A de la oscilación satisfará: ½ kA2 = ½ mv2’2 = ½ m(v2 – vcm)2 = ½ m(v2 – v2/2)2. Por tanto: ½kA2 = ½ m(v2/2)2 = ½ mvo2/(1 + γ)2 , donde se tuvo en cuenta (3). Entonces: A = [(m/k) vo2/ (1 + γ)2]1/2 = vo/ [ω (1 + γ)]

(6)

La ley de movimiento de la bola 2 es entonces: x2 = xcm + x2’= (xcm,0 + vcmt) + (A sen ωt – xcm,0). De aquí: x2 = vot / (1 + γ) + vo/ [ω (1 + γ)] senωt (7) Si las bolas chocan de nuevo se cumplirá: x1 = x2, y teniendo en cuenta (4) y (7): v0 t (1 – γ) / (1 + γ) = vot / (1 + γ) + vo/ [ω (1 + γ)] senωt Arreglando algebraicamente: sen ωt = −γωt

(8)

Llamando θ = ωt nos queda: senθ = −γθ. Si llamamos y1 = sen θ y y2 = −γ θ, la ecuación (8) corresponde gráficamente a la intersecy ción entre la sinusoide y1 y la recta y2, como muestra el gráfico de la Fig. 17.2S. π 3π/2 Lo que queremos es las bolas 1 y 2 θ choquen al menos una vez más después del primer encuentro en el origen. Esto y2= -γz significa que la recta y2 debe cortar a la Fig. 17.2S y2*= -γz sinusoide en al menos un punto. Del gráfico 17.2S se ve que esto ocurre solamente si la recta se inclina como máximo hasta pasar muy cerca del mínimo de la sinusoide, donde θ = 3π/2. Si la pendiente de la recta se hace más vertical, no habrá otra intersección entre la sinusoide y la recta, por lo que no sucederá de nuevo que x1 = x2, y no habrá otro choque. Entonces, como máximo debe cumplirse, aproximadamente: sen(3π/2) = −γ3π/2 de donde resulta γ ≈ 0.21 como valor tope posible. Así pues, ocurrirá al menos un choque más si la relación de masas entre las bolas satisface: m/M < 0.2 166

17.3-El pistón deja de moverse cuando se evapora toda el agua: cuando el número de moles en estado gaseoso sea constante en cada parte, el aumento de presión y temperatura ocurrirá a volumen constante; en tal situación, el aire redujo su volumen de Vo a Vo/2 y el vapor lo aumentó de Vo a 3Vo/2. La evaporación del agua ocurrirá todo el tiempo en equilibrio de la fase líquida con su vapor saturado. Sean To y Po la temperatura y la presión al inicio, y T1, P1, los valores finales. Para el aire se cumplirá: PoVo/To = P1Vo/(2T1)

(1)

Para el vapor de agua, tratado como gas ideal, se tendrá al inicio: PoVo = (M/μ)RTo con Po = Ps (a temperatura To)

(2)

Y al final: P1(3Vo /2) = [(M+m)/μ]RT1 con P1 = Ps (a temperatura T1)

(3)

donde μ es la masa molar del agua y m= 4 g es la masa evaporada. De (1): PoVo = P1 (Vo/2)(To/T1). Sustituyendo en (2) y cancelando To: P1Vo/(2T1) = (M/μ)R. Dividiendo miembro a miembro por (3) se obtiene: 1/3 = M / (M+m), y M = m/2 = 2 g, que es la masa de agua al inicio. Para averiguar las temperaturas To y T1, así como la masa Mo de aire, necesitamos hacer el gráfico de Ps vs T, mostrado en la Fig. 17.3S. En ese gráfico hay dos puntos cruciales para nuestro análisis: el de la presión de vapor al inicio y el de la presión de vapor al final. Para estos dos puntos se cumplirá, por la ley de los gases ideales: Ps = (MR/μVo)T = 923T

y

Ps = [(M+m)2R/3μVo]T = 1848T

con Ps en Pa, T en K, y μ=18 g/mol para el agua. Estas ecuaciones representan dos líneas rectas de PsvsT. Si las expresamos en grados celsius, con T = t + 273, estas ecuaciones se convierten en: Ps = 252×103 + 923t

y

Ps =504×103 + 1848t

con Ps en Pa y t en grados celsius. O, si lo expresamos en kPa:

Ps = 252 + 0.923t

y Ps = 504 + 1.85t.

Estas dos rectas se han representado en el gráfico de la Fig. 17.3S (próxima página). Para trazarlas, se determinaron sus intersectos con el eje de ordenada (252 kPa y 504 kPa) y se evaluó la presión para t = 100oC (344 kPa y 689 kPa), con lo que determinamos dos puntos de cada recta.

167

Ps(kPa)

Los dos puntos cruciales que nos interesan deben pertenecer a la curva de la presión del vapor de agua y a cada una de las dos rectas analizadas, lo cual corresponde a los puntos A y B del gráfico.

1000 800 600

689

B

504

400

A

380

344

200 252 0 0

El punto A nos dice que la tempera50 200 o tura inicial era de unos 140oC, con una t( C) presión de unos 380 kPa. El punto B Fig. 17.3S nos informa que la temperatura final era de unos 170oC. O en kelvin: To = 413 K y T1 = 443K. Entonces, para el aire seco conocemos que To = 413 K y Po = 3.8×105 Pa, con μ = 29 g/mol. Por tanto la masa de aire seco era: Mo = PoVoμ/(RTo) = 3.2 g. 100

140

150

170

17.4-En la Fig. 17.4Sa se ha representado el gráfico de T vs t. Para responder a las preguntas formuladas es más conveniente un gráfico de velocidad de calentamiento a cada temperatura. Así, para varios puntos del gráfico original podemos trazar la recta tangente y medir su pendiente, que nos indicará la velocidad de calentamiento, en oC/min, para cada temperatura del agua. Se obtiene así el gráfico de la Fig. 17.4Sb (se graficaron sólo cuatro puntos). ΔT/Δt(oC/min)

T(oC) 60

6.5

6

40

4

20

2

0

3.5

0 0

10

5

t(min)

0

20

Fig. 17.4Sa

40

T(oC)

60

80 Fig. 17.4Sb

(El segundo gráfico también se puede construir a partir de los datos de la tabla, restando cada dos temperaturas consecutivas y dividiendo por el intervalo de 1 min que los separa. Este cociente representará la velocidad de calentamiento en la mitad del intervalo de temperaturas (aproximación lineal). Se obtiene así la siguiente tabla que sirve también para hacer el segundo gráfico). t(min) 0.5 ΔT/Δt 6.2 (oC/min) T(oC) 23.1

1.5 5.6

2.5 5.0

3.5 4.6

4.5 4.2

5.5 3.7

6.5 3.4

7.5 3.1

8.5 2.7

9.5 2.6

29.0

34.3

39.1

43.5

47.5

51.0

54.3

57.2

59.8

168

Del gráfico se ve que la velocidad de calentamiento se hace cero cerca de los 85oC, de modo que no se calentará hasta hervir. Ya cerca de los 85oC el agua cede calor al ambiente con la misma velocidad con que lo recibe de la resistencia. Esto responde a la segunda pregunta. En cuanto a la primera pregunta puede observarse del gráfico de calentamiento que a la temperatura ambiente, de 20oC, la velocidad de calentamiento es 6.5 oC/min, y a 50oC la velocidad de calentamiento se reduce a 3.5oC/min. La disminución de la velocidad de calentamiento es de 3.0oC/min, y se debe a la diferencia entre el calor que retiene el agua a temperatura ambiente y el que retiene a 50oC. O sea, tal diferencia es la debida al calor que entrega el agua caliente al ambiente cuando está a 50oC. Entonces, si se apaga la resistencia cuando el agua está a 50oC, la velocidad con que se enfriará será de 3.0oC/min, lo que significa que en 1 minuto se enfriará unos 3oC.

17.5-El calor absorbido por el agua es rápidamente repuesto a lo largo del tubo gracias a la gran conductividad de éste en relación a la del agua. En el estado estacionario se cumplirá la proporcionalidad: Th/Ts = h/l (1) donde h es la altura a que sube el agua en el capilar y Th es la temperatura a esa altura. La fuerza de tensión superficial que levanta a la columna líquida es: F = 2πrζ cosα, donde α = 0 pues el líquido moja perfectamente al capilar (es el ángulo de contacto líquido-capilar). Esta fuerza equilibra a la de gravedad de la columna líquida, Fg = mg = πr2hρg. Así: 2πrζ = πr2hρg, de donde resulta: h = 2ζ/(ρgr) (2) Despejando h de (1) y sustituyendo en (2): l Th/Ts = 2ζ/(ρgr) , y ζ = ρgr l Th/(2Ts)

(3)

que nos da el valor de σ en función de Th, que es una recta de pendiente ρgrl/(2Ts) = 0.78×10-3 N/moC. 80

σ(×10-3 N/m)

60 40 20

Fig. 17.5S 20

40

60

80

100

T(oC)

Llevando esta recta a un gráfico junto con la curva de ζ vs T que sale de la tabla (Fig. 17.5S), se obtiene un punto de intersección que nos da el valor del coeficiente de tensión superficial ζ a la 169

altura de la columna líquida y la temperatura a esa altura del agua: cerca de 60×10 -3 N/m y cerca de 80oC. La altura podemos evaluarla por (1): h = l Th/Ts = 8×80/100 ≈ 6.4 cm. (O por (2): h = 2ζ/(ρgr) = 2×60×10-3/(1×103×9.8×2×10-4) ≈ 6.1 cm)

T(oC) 3

17.6-Del gráfico se desprende que en los primeros 50 min en la habitación el hielo se estaba derritiendo con el calor ganado de la habitación y el sistema no cambió de temperatura (permaneció en 0oC). De los 50 min en adelante, el sistema era todo líquido y comenzó a calentarse con el calor recibido de la habitación. Durante 10 min (de 50 min a 60 min en la Fig. 16.6S) la mezcla subió 2oC y el calor absorbido fue:

2 1

20

Fig.17.6S

Q = MmezclacΔT = 10×4.2×2 = 84 kJ

40

60

t(min)

La habitación tiene unos 20 o 30oC, de modo que la diferencia de temperaturas no cambia apreciablemente entre la mezcla y el aire de la habitación durante todo el proceso (cambia 2 grados de 20 o 30) por lo que la velocidad de intercambio de calor entre la mezcla y la habitación puede asumirse prácticamente constante durante los 60 min iniciales. En tal caso, el calor intercambiado en 50 min es cinco veces mayor que el intercambiado en 10 min, esto es: Q’= 5Q = 420 kJ. Pero todo este calor fue empleado para derretir hielo, sin calentar la parte líquida. Entonces, la masa fundida de hielo es: Mhielo = Q/Lf = 420/340 ≈ 1.2 kg de hielo

17.7- Llamemos q al calor desprendido por unidad de tiempo de la hornilla, y: C=200 J/K (capacidad calorífica de la cazuela), ca = 4.18 kJ/kg.K (calor específico del agua), cal (calor específico del alcohol), ma= ρaV = 0.50 kg (masa de agua), mal = ρalV = 0.56 kg (masa de alcohol). En ambos casos los líquidos comienzan a calentarse a la temperatura de 20oC, según se desprende del grafico. Consideraremos el calor desprendido en un tiempo Δt pequeño, próximo a los 20oC. En ambos casos, esta cantidad de calor será Q = qΔt, y se empleará en calentar al líquido (mcΔT), calentar la cazuela (CΔT) y en algunas pérdidas al exterior. 170

T(oC) 100 80 00 60 40 20 t(min) 10

20

30

Fig. 17.7S

40

50

Estas pérdidas al exterior dependen de la diferencia de temperaturas entre el sistema y el exterior (T – 20), el tiempo de calentamiento Δt, y factores geométricos y ambientales (corrientes de aire, dimensiones de la superficie de la cazuela, etc.) que agruparemos en una constante α. Entonces, para el agua: qΔt = macaΔTa + CΔTa + α (T – 20)Δt

(1)

Y para el alcohol: qΔt = malcalΔTal + CΔTal + α (T – 20)Δt

(2)

Hemos tenido en cuenta que el agua y el alcohol se calientan desigualmente (ΔTa, ΔTal) en un mismo tiempo Δt. Considerando un mismo tiempo de calentamiento inicial, qΔt es el mismo en ambos casos, por lo que: macaΔTa + CΔTa + α (T – 20)Δt = malcalΔTal + CΔTal + α (T – 20)Δt Considerando un Δt pequeño (como 1 min), la temperatura T alcanzada no diferirá grandemente, por lo que el último sumando de cada lado valdrá casi lo mismo y se cancelan uno con otro: macaΔTa + CΔTa = malcalΔTal + CΔTal Dividiendo ambos miembros por el tiempo Δt transcurrido: macaΔTa/Δt + CΔTa/Δt = malcalΔTal/Δt + CΔTal/Δt De aquí despejamos cal: cal = {[(ΔTa/Δt) / (ΔTal/Δt)] [(maca + C)/mal]} – C /mal Son conocidos ma, mal, ca y C; los cocientes ΔTa/Δt y ΔTal/Δt) son las pendientes de las curvas en las proximidades del instante inicial, que pueden determinarse a partir del gráfico (6.8oC/min para el alcohol y 4.4oC/min para el agua). Sustituyendo valores: cal = 2.4×103 J/kg.K. Llamando al calor de ebullición del alcohol, Lal, cuando el alcohol haya hervido un tiempo Δt1, se cumplirá: qΔt1 = Lal Δm + α (Tal – 20) Δt1, donde Tal = 78oC (del gráfico) y se tuvo en cuenta que la cazuela no se calienta durante la ebullición. De aquí: Lal = [q – α (78 – 20)] / (Δm / Δt1)

(3)

El denominador es la rapidez de ebullición, y puede calcularse a partir del dato de que la mitad del alcohol ebulle en 23 min: Δm / Δt1 = (0.56/2)/23 = 1.2×10-2 kg/min. Y el corchete con q y α juntas pueden evaluarse a partir de la ecuación (1), aplicando los datos de la curva del agua a 78oC (en t = 13 min): q – α (78 – 20) = (maca + C) ΔT/Δt = (0.50×4180 + 200) (78 – 20) / 13 min = 1.02×104 J/min Sustituyendo en (3): Lal = [1.02×104 J/min] / (1.2×10-2 kg/min) = 8.5×102 kJ/kg 171

17.8-La erupción comienza apenas la presión del vapor saturado del agua en la base del canal sobrepase ligeramente a la suma de presión atmosférica con la presión hidrostática de la columna de agua en el canal de salida, bajo una profundidad de 90 m. O sea, se produce la erupción cuando P ≈ Patm + ρgh ≈ 0.98×10 6 Pa ≈ 106 atm Del grafico se ve que para esa presión de vapor la temperatura es de 180oC. El calor de la tierra ha elevado la temperatura del agua subterránea hasta esa temperatura, con la cual la presión de vapor empieza a superar a la presión exterior que oprime al agua, formándose las burbujas de vapor que empiezan a subir por el canal por flotación: el geiser expulsa vapor de agua que arrastra algo de agua líquida. En la medida que sale vapor sobrecalentado se va enfriando el agua en el canal hasta que su temperatura baja hasta los 100oC y deja de hervir incluso el agua que está cerca de la salida de la canal del geiser. En el proceso, la temperatura del agua del reservóreo baja desde los 180oC hasta unos 100oC, y toda el agua M del reservóreo pierde una cantidad de calor McΔT, con ΔT = 80oC. Ese calor perdido por el agua del reservóreo es el empleado en hacer hervir una masa m de agua que se expulsa como vapor al exterior, calor que vale Lvm. Entonces: McΔT = Lvm De aquí:

m/M = cΔT/Lv = 4180×80/2260000 = 0.15

O sea, en cada erupción el reservóreo pierde alrededor del 15% de su agua. (Al bajar la presión en el reservóreo entra más agua de las fuentes subterráneas de alimentación y tras un tiempo se calienta lo suficiente para repetir otra erupción, y así lo hace cíclicamente).

0.8

17.9-De acuerdo con la potencia y el voltaje de trabajo de cada lámpara, sus corrientes eléctricas a 110 V son:

0.6

I1 = P1/V1 = 60/110 = 0.545 A y

I(A)

I2 = P2/V2 = 100/110 = 0.909 A Fig.17.9S

0.4 0.2

20

40

60

80 V (V)

Al colocarlas en serie, la corriente debe ser la misma en ambas lámparas, y sus voltajes deben sumar 220 V. Si la de 60 W no admite una sobrecarga mayor del 10% que la que admite a 110 V, su máxima corriente sin quemarse será de: 0.545 + 0.054 = 0.599 A ≈ 0.6 A. 172

Con tal corriente la lámpara de 100 W tendría un voltaje de 50 V (Fig. 17.9S), que sumado a los 110 V de la primera lámpara quedaría muy por debajo de los 220 V aplicados. Significa que la lámpara de 100 W consumirá mayor voltaje que 50 V y permitirá el paso de más de 0.6 A. con lo cual se quema la lámpara de 60 W.

17.10- a)Si se conectan en serie va a pasar la misma corriente por ambas lámparas y caerá el I(A)

(c1)

0.30

(b1) (a)

0.20

(b2)

(c1)

(c2)

(b1) (a)

0.10

Relación I-V para la lámpara (Resistencia no lineal)

(b2) (c2) 1.0

2.0

3.0

4.0

V(V)

Fig. 17.10S

mismo voltaje en ambas. O sea, caerán 2.5 V en cada una, y por la gráfica se ve que a ese valor de voltaje corresponde una corriente de 0.27 A (Ver líneas (a) de la Fig. 17.10S). b)Si en una lámpara caen 3.5 V, en la lámpara que esté en el paralelo con la resistencia cae la diferencia 5.0 – 3.5 = 1.5 V. Por la gráfica, la lámpara con 3.5 V tiene una corriente de 0.29 A (líneas b1); y la que está en el paralelo, con 1.5 V, tiene una corriente de 0.22 A (líneas b2). La diferencia entre estas dos corrientes es la que pasa por la otra rama del paralelo donde está la resistencia: I = 0.29 - 0.22 = 0.07 V. Significa que esa resistencia tiene un valor de: R = V/I = 1.5/0.07 = 21Ω c)Si conectamos dos lámparas en paralelo con una tercera en serie, la que está sola en serie soportará una corriente doble de las otras dos, y ésa sería la primera que se quemaría. Esto ocurriría si el voltaje de la lámpara en serie alcanzara los 4.0 V, con una corriente de 0.30 A (líneas c1). En cuyo caso, las otras dos lámparas tendrían una caída de voltaje de 5.0 – 4.0 = 1.0 V. Pero de acuerdo con la gráfica, cada lámpara del paralelo bajo 1.0 V de voltaje debe soportar una corriente de 0.17 A (líneas c2), y ambas juntas consumirían 0.34 A. Pero si “halan” juntas más corriente que la de 0.30 A, se funde la primera lámpara. Por otro lado, si asumimos que en la 173

lámpara en serie caiga un voltaje menor de 4.0 V, entonces en las dos lámparas en paralelo caería un voltaje mayor de 1.0 V, con más corriente aún que 0.17 A, por lo que la lámpara en serie se quemaría más rápido. Por tanto, no pueden colocarse dos en paralelo con otra en serie pues se quemaría la que está sola en serie.

17.11-Llamemos V al voltaje de la fuente, Vl al de la lámpara y VR al de la resistencia. Se cumplirá: V = Vl + VR = Vl + IR

(1)

De aquí: I = V/R – Vl/R

I(A)

que es la ecuación de una recta en el plano de I vs Vl (la “línea de carga”) (Fig. 17.11S). Su intercepto con el eje de las corrientes será para Vl = 0, en cuyo caso I = V/R = 10/4 = 2.5 A. El intercepto con el eje de los voltajes será cuando I = 0, en cuyo caso: Vl = V = 10 V. Con estos dos puntos se puede trazar la línea de carga como muestra el gráfico, la cual fija el punto de operación de la lámpara (intersección de la línea de carga con la curva de la lámpara), pues en tal punto la corriente satisface los valores de VR, Vl y V que cumplen con la ecuación (1).

2.0

1.0

5

10

Vl (V)

Fig. 17.11S

La corriente de operación de la lámpara es entonces I ≈ 1.1 A y su voltaje de operación es Vl ≈ 5.5 A. La potencia de la lámpara es entonces: P = IVl ≈1.1×5.5 ≈ 6 W

17.12- Entre los puntos A y B no hay ningún elemento. Son sólo dos puntos entre los cuales se produce cierta diferencia de potencial, medible por un voltímetro ideal (de corriente nula). De tal forma, la misma corriente que pasa por r es la que pasa por la lámpara L. (a)Sea V el voltaje en la lámpara L. Se cumplirá: E = V + Ir, de donde: I =E /r – V/r = 0.4 – 0.1V. En un gráfico de I vs V ésta es una línea recta (“línea de carga”) de pendiente 1/r. Esta línea nos informa cómo depende el voltaje en la lámpara de la corriente que pasa por la resistencia r en serie con la lámpara. En la Fig. 17.12Sa hemos representado la línea de carga de la resistencia r superpuesta a la característica voltampérica de la lámpara. El punto de intersección de ambas líneas fija el “punto de operación” de la lámpara (su corriente y su voltaje para la resistencia r dada).

174

I(A) 0.4 0.3

0.4 0.3

0.24

ΔI

0.2 0.1 0

0.2 0.1 0.08

1.6

0

1

2

3

4

5

V(V)

0

0

Fig.17.12Sa

1

ΔV 2

3 3.4 4

5

V(V)

Fig.17.12Sb

Este punto de operación es de V =1.6 V e I = 0.24 A según se puede apreciar en la Fig. 17.12Sa. La resistencia de la lámpara, en corriente directa será RL = V/I = 1.6/0.24 ≈ 6.7 Ω. (b)Para que entre A y B no haya diferencia de potencial, el reóstato, o potenciómetro, debe quedar dividido en dos resistencias, R1 (parte de abajo) y R2 (parte de arriba) tales que: R1/R2 = V/ (E – V). De aquí: R1= (R1+R2) V/E = RV/E = 16 Ω, y en consecuencia: R2 = R – R1 = 24 Ω. (c)Bajo pequeñas variaciones de la fem se producen pequeñas variaciones de voltaje en todos los elementos del circuito. Para que las variaciones de voltajes en AB, ΔVAB, sean mínimas, la variación de voltaje en L deberá ser casi igual a la variación de voltaje en R1 (parte baja de R). La resistencia R1 es constante, pero RL depende del valor del voltaje V aplicado a ella. Para pequeñas variaciones ΔV alrededor del punto de operación de la lámpara se producirán variaciones ΔI proporcionales a ΔV: ΔI ΔV (aproximación lineal para variaciones pequeñas). La constante de proporcionalidad será la pendiente de la recta tangente a la curva de la característica voltampérica de la lámpara en el entorno del punto de operación (Fig. 17.12Sb). En consecuencia, bajo pequeñas variaciones de voltaje ΔV, cerca del punto de operación, la lámpara se comporta como una resistencia de valor rL = ΔV/ΔI = cot θ, donde θ es el ángulo de inclinación (analítico) de la recta tangente a la curva. (Ésta es la resistencia dinámica, o diferencial, de la lámpara, bajo corrientes variables). Esta resistencia no relaciona los valores de V e I en el punto de operaciones (eso lo hace RL), sino que relaciona las variaciones de V con las de I. En nuestro caso: rL = (3.4 – 0)/(0.40 – 0.08) ≈ 11 Ω

(ver valores en el gráfico, Fig. 17.12Sb)

Ahora, la variación ΔV en la lámpara satisface: ΔV =ΔE − r ΔI, donde ΔI = ΔV/rL. Queda entonces: ΔV =ΔE − ΔV(r/rL). Y despejando: ΔV = ΔE rL/(r+rL)

(1)

Por el divisor de voltaje teníamos: V1 = E R1/R, por lo que ΔV1 = ΔE (R1/R)

(2)

Deseamos que ΔV = ΔV1 para que el ΔVAB = 0. Entonces, igualando (1) con (2): 175

ΔE rL/(r+rL) = ΔE (R1/R). De aquí: R1 = RrL/(r + rL) = 21 Ω. O sea, el punto móvil del reóstato debe estar en una posición en la que R1 = 21 Ω. En esta condición: V1 = E R1/R = 2.1 V y V = 1.6 V (punto de operación), por lo que VAB= 0.5 V. Si hubiera una fluctuación de 1 V en la batería, ΔE = 1 V, los cambios en L y R1 serían: ΔV = ΔE rL/(r+rL) = 0.524 V y ΔV1 = ΔE (R1/R) = 0.525 V, por lo que la fluctuación de voltaje entre A y B sería ΔVAB = 0.525 – 0.524 = 0.001 V. Así con una variación de un 25% de la fem (1 V en 4 V), la variación del voltaje entre A y B sería del 0.2% (0.001 V en 0.5 V). Este circuito puede ser usado como un estabilizador de voltaje.

17.13-El voltaje que cae en el diodo, Vd, será:

I(mA) 1.45 1.5 1.63

Vd = V – IR1 – IR2, por lo que: I = V / (R1+R2) – Vd/ (R1+R2)

T1 T2 T3 T1

1.15

(1)

0.69 0.60

Esta ecuación representa una recta, I = f(Vd), en el gráfico de I vs Vd, de pendiente:

1.0

T2

0.5

T3

0.20

1.0

0.4

0.8

1.2 Vd(V)

m =−1/(Ri+R2) e intercepto b=V/(R1+R2); Fig. 17.13S esta recta es la “línea de carga”. La intersección de esta línea de carga con la curva voltampérica fija el punto de operación del diodo. De la ecuación (1) se ve que la pendiente de la línea de carga es independiente de la temperatura T, pues solamente depende de las resistencias, que no nos dicen que dependan de la temperatura. El intercepto en el eje de las I sí cambia pues depende de V, que es función de la temperatura. Así, a distintas temperaturas la línea de carga se desplaza arriba y abajo, paralelamente a su inclinación original. Se conoce por dato que ΔV/ΔT = 2.5×10-2 V/oC = α y que V2 = 6.0 V a T2 = 25oC. Entonces, a T1= 125oC se tendrá: V1= V2 + α(T1 – T2) = 8.50 V y a T3 = -60oC se tendrá: V3= V2 + α(T3 – T2) = 3.87 V. Se ve que el voltaje de la fuente varía fuertemente con la temperatura en el intervalo de interés. Busquemos el intercepto de la línea de carga con el eje I, b=V/(R1+R2), para cada temperatura: b1= 1.63 mA, b2= 1.15 mA y b3= 0.69 mA. La pendiente en todos los casos es la misma: m = 1/(5.0+0.2) = 0.192 kΩ-1 ≈ 0.2 mA/V Las tres líneas de carga se muestran aproximadamente en la Fig. 17.13S. Se trazó primero la recta inferior de pendiente -0.2 (pasa por el eje I en I = 0.2 mA y por el eje Vd en Vd = 1 V). 176

Después se construyen las tres rectas paralelas a la inferior, pero con sus respectivos interceptos en el eje I. Operando a 25oC, se ve del gráfico que I = 1.0 mA y Vd = 0.80 V. La caída de voltaje en la salida AB será: VAB = V – IR1 = 6 – 0.001×5000 = 1 V, que es la respuesta a la primera pregunta. Cuando la temperatura varíe entre -60oC y 125oC, las variaciones de voltaje en la salida AB se calcularán como: ΔVAB = ΔV – (ΔI) R1 (2) Del gráfico se ve que la variación de I entre -60oC y 125oC es ΔI = 1.45 – 0.60 = 0.85 mA. La variación por cada grado celsius será: ΔI/ΔT = 0.85/(125 + 60) = 0.0046 mA/ oC. Si dividimos la ecuación (2) por ΔT tendremos: ΔVAB/ΔT = ΔV/ΔT – (ΔI/ΔT) R1 = α – (ΔI/ΔT) R1 Para el intervalo total de temperaturas: ΔVAB/ΔT = 2.5×10-2 V/ oC − 0.0046 (mA/ oC) × 5(kΩ) = 0.002 V/ oC Ésta es la variación del voltaje de salida por cada grado centígrado que cambie la temperatura. Asumiendo una variación lineal del voltaje de salida con la temperatura podremos plantear: VAB (T)= VAB(25oC) + (ΔVAB/ΔT ) (T – 25). O sea: VAB (T)= 1 + 0.002 (T – 25) con T en grados celsius. Ésta es la respuesta a la segunda pregunta.

177

18-Problemas con Análisis de Simetría.

C

18.1-Encuentre la capacitancia del sistema de capacitores idénticos de la Fig. 18.1. La capacitanFig. 18.1 cia de cada capacitor es C.

C

C

R

18.2-Determine la resistencia del circuito RAB entre los puntos A y B de la estructura compuesta por los nueve alambres iguales mostrados en la Fig. 18.2, de resistencia R cada uno.

R

R

B

R R

R

A

C

C

C

R

R

R

Fig. 18.2

C

18.3-En el circuito de la Fig. 18.3 todos los resistores tienen la misma resistencia R. La tensión en los bornes es V. Determine la intensidad de la corriente I en los cables de alimentación del circuito. La resistencia de los cables de alimentación es despreciable.

R R

A

R

R B

R

E

R

R

R V

A R1

kR1 R2

kR2

B Fig. 18.4

D

Fig. 18.3

18.4-El circuito de resistores de la Fig. 18.4 consiste en un número de eslabones muy grande. La resistencia de los resistores de cada eslabón es k veces mayor que el valor de las resistencias en el eslabón anterior. Encuentre la resistencia RAB entre los puntos A y B si las resistencias del primer eslabón son R1 y R2.

B

18.5-La arista de cada cuadrado de la Fig, 18.5 es un alambre de resistencia r = 10 Ω. En el cuadrado central se suelda una lámina de un material muy buen conductor (área sombreada). Hallar la resistencia entre los puntos A y B. Fig. 18.5 178

A

18.6-(Requiere Cálculo) Un fino alambre muy largo posee una densidad lineal de carga λ constante en toda su extensión. Calcule la intensidad del campo eléctrico a una distancia r del alambre, lejos de sus extremos y próximos al alambre. Calcule también la diferencia de potencial entre dos puntos a las distancias r1 y r2 del alambre.

18.7-(Requiere Cálculo) Una esfera metálica de carga q1 y radio r1 está en el interior de una fina capa metálica esférica de radio r2 y carga q2; ambas esferas son concéntricas. Calcule la intensidad del campo eléctrico a diferentes distancias del centro de ambas esferas.

18.8-A una esfera conductora homogénea de masa m se le transfiere una carga q y se fija al extremo de un hilo no conductor. El otro extremo se liga al extremo superior de un anillo de radio R, el cual se halla en un plano vertical. Al anillo, elaborado de un material conductor, se le ha comunicado una carga Q del mismo signo que la carga q, que se distribuye uniformemente por el anillo. Determine la longitud l del hilo con la cual la esfera se sitúa justamente sobre el eje horizontal del anillo.

18.9-(Requiere Cálculo) Calcule el valor de la inducción magnética, B, en el interior de una bobina cilíndrica muy larga (solenoide) de n vueltas por unidad de longitud y sin núcleo ferromagnético, cuando circula por ella una corriente eléctrica de intensidad I.

18.10-(Requiere Cálculo)Calcule el valor de la inducción magnética, B, en el interior de un toroide de sección circular que tiene un enrollado uniforme de n vueltas por unidad de longitud, por el cual circula una corriente de intensidad I. Se conocen el radio de la sección circular, r, y el radio del toroide, R, cumpliéndose que R >> r. Haga el cálculo asumiendo que el toroide está vacío e indique cómo variaría el resultado si el toroide estuviese lleno de un material ferromagnético. (Toroide: es como una dona, o salvavidas, en la que se enrolla el cable en vueltas muy apretadas).

179

18-Problemas con Análisis de Simetría. Soluciones.

18.1-En la Fig. 18.1Sa hemos numerado los capacitores, aunque son todos iguales, para representar el circuito de una manera más simétrica sin perder de vista cuál es cada capacitor en el nuevo circuito equivalente. La Fig.18.1Sb presenta un circuito equivalente al dado. En este nuevo circuito puede notarse con facilidad que el capacitor C3 está conectado a puntos de igual potencial y que no se cargará, lo que es equivalente a quitarlo del circuito sin que se afecte la distribución de cargas en el resto.

C1

C1

C2

C3

C

C

C

C

C

C4 C3

C6

C6

C5

C4

C2

Fig. 18.1Sa

C5

Fig. 18.1Sb

Fig. 18.1Sc

Pero suprimir a C3 equivale a tener el circuito de la Fig. 18.1Sc, que es tratable como un conjunto de series y paralelos bien definidos. Son tres ramas en paralelo con capacitancias C, C/2 y C/2 que sumadas dan una capacitancia total de 2C.

18.2-En este problema hay nueve ramas con diez corrientes diferentes si se incluyen la de enR R A

R R

B

R R

R R

R Fig. 18.2

I5

F I1

I3

A

I2

I0

E I4 D I5

I2 I3

B I0 I1

C

Fig. 18.2S

trada y la de salida, I0. En la Fig. 18.2S, la recta DE divide la estructura en dos partes simétricas, con distribuciones equivalentes de resistencias, voltajes y corrientes, lo cual reduce las incógnitas a seis: I0, I1, I2, I3, I4, I4 e I5, y se simplifican mucho los cálculos. Se han representado las corrientes que son iguales en las distintas ramas simétricas. Aplicando ley de nodos en F: I1 = I3 + I5 Aplicando ley de nodos en D: I2 + I3 = I4 + I5 Aplicando ley de mallas FDEF: (I3+I4) R = I5R Aplicando ley de mallas AFDA: (I1+I3) R = I2R 180

Resolviendo estas cuatro ecuaciones en función de I1: I2 = (6/5) I1, I3 = (1/5)I1, I4 = (3/5)I1, I5 = (4/5)I1. Añadiremos ahora una ecuación de ramas desde A hasta B para evaluar la caída de potencial VAB, necesaria para calcular la resistencia RAB, que será RAB = VAB/I0, donde I0 = I1+I2 (ver nodo A). O sea: VAB= [I1 + (4/5) I1+ (6/5) I1] = 3I1R, de donde resulta VAB /I1 = 3R. Ahora: RAB = VAB/I0 = VAB/ (I1+I2) =VAB/[I1+ (6/5)I1] = (5/11)VAB/I1 = (15/11)R Así: RAB = (15/11) R.

18.3-En la Fig. 18.3 se han añadido letras para identificar los nodos y las resistencias. Puede notarse que las mallas ABCA y ABDA son equivalentes, por lo que los puntos C y D son equipotenciales, que es como si estuvieran conectados entre sí (sin resistencias de por medio). C R

R R

A R

C,D R

B

R

R/2 E

A

R/2 R

B

R/2

R

E

R R

V

D

V

Fig. 18.3

Fig. 18.3S

La Fig. 18.3S muestra el circuito equivalente, donde C y D se han unido en un solo punto, y es como si las resistencias AC y AD quedaran en paralelo, así como BC con BD y EC con ED. Visto así el circuito es uno sencillo de series y paralelos, con una rama BEC de resistencia (3/2)R en paralelo con la rama BC de resistencia R/2, todo esto en serie con la rama AC de resistencia R/2, y todo en paralelo con la rama AB de resistencia R. Así: 1/RCB = 1/(R/2) + 1/(R + R/2), de donde: RCB = (3/8)R Entonces: RACB = R/2 + (3/8)R = (7/8) R. Finalmente: 1/RAB = 1/R + 1/[(7/8)R]. Y resolviendo: RAB = (7/15)R

181

18.4-En este circuito podemos mentalmente dividir por k todas las resistencias excepto las del primer eslabón, R1 y R2. Si hacemos esto nos damos cuentas de que lo que quedaría detrás del primer eslabón es un conjunto de resistencias igual que el dado entre A y B, de modo que la secuencia que prosigue tras el primer eslabón es la misma que la cadena completa multiplicada por k. Este análisis nos permite sustituir el circuito original por el equivalente de la Fig. 18.4S, que ya es un circuito sencillo de serie y paralelo, donde se cumplirá: RAB = R1 + [1/R2 + 1/kRAB]-1

A R1

Despejando RAB surge una ecuación de segundo grado:

R2

kRAB2 + RAB[R2(1 – k) – kR1] – R1R2 = 0

kRAB

B

Resolviéndola:

Fig. 18.4S

RAB = {kR1 – R2(1 – k) ±

}/(2k)

La solución negativa carece de significado físico.

18.5-La lámina conductora pone a un mismo potencial todos los alambres del cuadrado soldado a ella. La diagonal de la Fig.18.5Sa divide el circuito en dos ramas simétricas conectadas a los puntos de potencial A y de potencial B, por lo que equivalen a dos ramas en paralelo. Por la simetría de las dos ramas, los puntos C y D están al mismo potencial, por lo que las dos aristas de la esquina inferior no conducen corriente entre C y D y pueden suprimirse en el análisis. Los lados 1, 2, 3, 4, 5, 6, 7, 8, 9 son simétricos a los lados 1’, 2’, 3’, 4’, 5’, 6’, 7’, 8’, 9’. Calcularemos la resistencia equivalente de las nueve primeras resistencias, igual a la de las otras nueve, y luego calcularemos la equivalente de ambas en paralelo.

5

3

6

1 2

4

B 1’

7 8 C

2’

A 9 7’

9’ D

8’

3’

4’ 5’ 6’

Fig. 18.5Sa

A continuación (Fig. 18.5Sb) se muestra el esquema del circuito equivalente del grupo 1, 2, 3, 4, 5, 6, 7, 8, 9, reducido a otros circuitos equivalentes cada vez más simples. Las resistencias equivalentes que se escriben en cada paso se calculan por las operaciones de dos resistencias equivalentes en serie o en paralelo: Rserie = Ra + Rb y Rparal = RaRb/(Ra + Rb) El circuito se ha dibujado partiendo del punto B y siguiendo todos los caminos que llevan al cuadrado central, A, que tiene un único potencial, simbolizado por la línea inferior del esquema de circuito de la Fig. 18.5Sb (como si fuera un potencial de “tierra” común). 182

r

Las resistencias 8 y 9 en serie equivalen a una de valor 2r.

B 8 r

6 r

9

3

5

7

1 2

4

A B

Ésta de valor 2r está en paralelo con la resistencia 7 de valor r, y equivalen a una de valor 2r/3.

6 r

2r

5

7

2

4

r

A B

r

1

3

(2/3)r

La de 2r/3 está en serie con las dos de valor r cada una, por lo que todas juntas equivalen a una de valor 8r/3.

1

3

4

2 A B 3

(8/3)r

1

r

2 A B 1

r

La de valor 8r/3 está en paralelo con una de valor r y equivalen a una de valor 8r/11.

(8/11)r

2 A

La de valor 8r/11 está en serie con una de valor r y equivalen a una de valor 19r/11. Ésta de 19r/11 está en paralelo con una de valor r y equivalen a otra de valor 19r/30. Finalmente, la de 19r/30 está en serie con otra de valor r y dan una resistencia equivalente total de 49r/30, para la rama 1, 2, 3, 4, 5, 6, 7, 8, 9.

1 (19/11)r

B

r A r

B

(19/30)r

Esta rama está en paralelo con la rama 1’, 2’, 3’, 4’, 5’, 6’, 7’, 8’, 9’, de igual valor, y las dos juntas dan una resistencia total R = 49r/60.

A B (49/30)r

Y como r =10 Ω, entonces R = 8.2 Ω

Fig. 18.5Sb

183

A

18.6-El tipo de campo que se genera desde un alambre cargado uniformemente, lejos de los extremos del alambre y cerca del mismo, es de líneas radiales como muestra la Fig. 18.6S. P r

Tapa 1

Tapa 2

Rollo

(a) Vista lateral

Fig. 18.6S

Vista en sección

(b)

Es un campo de simetría cilíndrica. Tomando una superficie gaussiana cilíndrica que contenga al punto P, que está a una distancia r del alambre como muestra la figura, podemos plantear la ley de Gauss: = q/εo, donde q es la carga neta dentro de la superficie gaussiana (o sea, la carga del alambre que queda dentro del cilindro gaussiano. Así: ∫tapa 1EdA + ∫tapa 2EdA + ∫rollo EdA = q/εo

(1)

Pero en las tapas del cilindro el flujo de campo es cero, y en la superficie enrollada el valor de E es el mismo en todos sus puntos, por lo que la última integral da EArollo = E(2πrl), donde l es la longitud del cilindro dibujado, y (2πrl) es el área de la superficie del rollo. Sustituyendo en (1): E2πrl = q/εo Ahora: q = λl, por lo que: E = λ/(2π εor), que es la intensidad del campo a la distancia r del alambre. En cuanto a la diferencia de potencial entre dos puntos a las distancias r1 y r2, basta recordar: E = − dV/dr, por lo que dV = −Edr = − [ λ/(2π εor)] dr. E integrando: ΔV = Esta integración da: ΔV = [λ/(2π εo)] ln(r1/r2)

18.7-El campo es radial, de simetría esférica, dado que las cargas están distribuidas uniformemente por las esferas metálicas, por lo que las superficies gaussianas apropiadas serán esferas de distintos radios r, acorde con las distintas zonas en que calcularemos la intensidad del campo (esferas gaussianas representadas por líneas quebradas en la Fig.18.7S, próxima página); en estas esferas el campo E es colineal con el dA en cada punto de la superficie, por lo que E.dA = E dA, y la E seá constante en cada integral. Aplicando la ley de Gauss a cada zona resulta: a) Ea = 0 en r < r1 184

puesto que no hay carga interior en ninguna esfera gaussiana que se tome en el interior de la esfera de radio r1, como la esfera A1 de la Fig. 18.7S. b)Eb = kq1 /r

2

en r1 < r < r2

puesto que una esfera gaussiana de radio r tomada concéntrica con la esfera metálica interior, pero con su superficie entre la esfera metálica y la capa esférica, sólo encuentra en su interior la carga neta producida por la esfera interior, q1. (Vea la esfera A2 de la figura)

r A3 r2

Ec r

A2

q2 r1 q1

r Ea=0

A1

Eb

Fig. 18.7S

c)Ec = k ( q1 + q2) /r2 en r > r2

puesto que una esfera gausssiana de radio r concéntrica con las dos metálicas, pero con su superficie exterior a ellas, encuentra una carga neta en su interior igual a la suma de q1 y q2. (Vea la esfera A3 en la figura).

18.8-La Fig. 18.8S muestra la situación planteada. Se han representado los datos q, m, Q y R, y la forma en que debe quedar el sistema en equilibrio. Se han indicado, además, el ángulo θ que debe formar el hilo con el eje horizontal, la fuerza de tensión T en el hilo, el peso mg de la esfera, y un elemento de carga δq del anillo que crea la fuerza de repulsión δF representada, cuyas componentes δFx paralela al eje de simetría y δFy perpendicular al eje, también se han representado. Debido a la simetría del anillo, habrá otro elemento de carga δq diametralmente opuesto al representado que ejercerá una fuerza δF igual a la anterior, con iguales componentes x y y, pero dirigida hacia arriba, lo que hará que las componentes verticales de las fuerzas eléctricas se anulen y solamente queden las componentes axiales, δFcosθ. Igualmente pasará con las restantes parejas de cargas δq del anillo que estén diametralmente opuestas. La resultante en el eje x de todas las fuerzas eléctricas será Fx =Σq δEi cosθ, donde δEi = k δqi / l2, por lo que: Fx = (kq cosθ / l2) Σ δqi = (k qQ / l2) cosθ

(1)

δq l T θ q, mδFx θ δFy mg

R Q

Fig. 18.8S 185

δF=qδE

Ahora bien, la resultante de fuerzas sobre la esferita debe ser cero, tanto en la vertical como en la horizontal: Tcosθ – Fx = 0 y Tsenθ – mg = 0. De estas dos ecuaciones se elimina T y se obtiene: Fx = mg cosθ/senθ

(2)

Igualando (1) con (2): (kqQ/l2) cosθ = mg cosθ/senθ Los cosenos se cancelan y el seno es: senθ = R/l Queda entonces: kqQR/mg = l3, y de aquí: l = (kqQR/mg)1/3.

18.9-La Fig. 18.9S muestra un corte de la bobina y sus líneas de campo magnético: es un campo uniforme en su interior (simetría plana), que se debilita hacia los extremos de la bobina y es prácticamente nulo por su borde exterior paralelo a la bobina. Se ha dibujado una línea cerrada de “circulación” en forma de rectángulo, con el lado ab paralelo a las líneas del campo del interior de la bobina, los lados bc y da perpendiculares a las líneas del campo en el interior de la bobina y están en un campo nulo por su parte exterior, y un lado, cd, en zona de campo nulo. La corriente entra en el plano de la figura por la parte inferior de la bobina, y sale por la superior en cada vuelta. d

c

l

corriente sale

N a

b corriente entra

Fig. 18.9S

Al aplicar la ley de Ampere a la trayectoria cerrada escogida, tendremos: = µoI, donde I representa la corriente neta que atraviesa la superficie cerrada de integración, que en nuestro caso es N veces la corriente suministrada a la bobina (pasan N vueltas en la longitud l del rectángulo de “circulación”). Entonces: = ∫ab B dl cos 0 + ∫bcB dl cos 90 + ∫cd0 dl + ∫da B dl cos 90 = Bl. Y de acuerdo con la ley de Ampere: Bl = µoNI. De aquí: B = µo(N/l)I. El cociente N/l representa la densidad de vueltas de la bobina (lo empaquetado que están las vueltas) y se representa por n = N/l. Entonces: B = µonI.

186

corriente entra

1 corriente sale

R 2r 2

B

18.10-La Fig. 18.10S muestra un corte del toroide (simetría circular), sus radios r y R, y la geometría circular de las líneas del campo dentro del toroide. Por fuera del toroide el campo es nulo, tanto para puntos muy distantes del centro del toroide como para puntos en la zona central, pues aplicando la ley de Ampere, = µoI, en circunferencias como la 1 y la 2, se tiene que la coriente neta en el interior de la circulación es cero, por lo que en la integral la inducción B deberá ser cero también.

Asumimos que la corriente sale del plano del papel por el borde interior del toroide y entra por su borde exterior, y que las vueltas del enrollado están mucho más apretadas que lo que aparecen en la figura. En este caso tomamos la línea de circulación como una circunferencia que coincida con una línea de campo magnético dentro del toroide. Como r